Sie sind auf Seite 1von 68

Nakon osam godina ponovo sa Vama!

ta je to GAMA?
GAMA je asopis za kreativne mlade matematiare,
fiziare i programere, ali i za one koji su dosad
mislili da to nijesu.

Kome je namijenjen asopis GAMA?


Svim uenicima gimnazija i ostalih srednjih kola u
Crnoj Gori, ali e u njemu poneto moi da proitaju
i mlai, a mnoge stvari e, svakako, zanimati i
starije!

Ko je zasluan za drugi broj asopisa


GAMA?

U prvom redu, kreativni uenici Gimnazije


Slobodan kerovi iz Podgorice, a zatim grupa
profesora entuzijasta iz iste kole kao i profesori
sa Prirodno-matematikog fakulteta, iz Zavoda za
kolstvo i Ispitnog centra.

ta su ambicije asopisa GAMA?


Prije svega da ga prihvatite kao najbolji asopis za
popularisanje matematike, fizike i informatike. U tom
cilju trudiemo se da sa itaocima, kako uenicima
tako i profesorima i nastavnicima, uspostavimo to
prisniju saradnju, ime emo iz broja u broj biti jo
bolji. To zaista nije teko jer matematika i fizika
su tako lijepe, korisne i neizbjene, toliko prepune
divnih ideja i neoekivanih rjeenja nerjeivih

a t i n ij e s r a m ota. problema da je dovoljno biti samo aktivni posmatra,


Ne z n uesnik i objektivni reporter, naravno uz veliku

t a je n e h t je t i znati. informatiku podrku.

Sramo
Sokrat
Dakle,
PIITE NAM!
KRITIKUJTE NAS ILI POHVALITE!
PITAJTE NAS!
PREDLAITE NAM!
REDAKCIJA

Napomena: Zbog finansijskih problema, koji naalost prate jedan ovakav asopis, nijesmo mogli da tampamo
asopis u proloj kolskoj godini. Zato skreemo panju itaocima da su odjeljenja uenika, autora lanaka,
vaea za kolsku 2007/08. godinu.
Sadraj: GAMA
asopis za mlade matematiare, fiziare i programere,
broj 2, februar 2009. godine
Izdava: Gimnazija Slobodan kerovi, Podgorica
Poznati matematiari
Za izdavaa: Radia eki
Pitagora 2
Glavni i odgovorni urednik: Mirjana Pjei, prof.
Jedan problem vie rjeenja
Urednik uenik: Rastko Pajkovi IIIi
Pitagorina teorema 4
Redakcija:
Anegdote 9
Boo Bakovi, prof. (urednik za informatiku)
Feljton Radovan Ognjanovi, prof. (urednik za fiziku)
Istorija matematike I dio 10 Boljevi Gordana, uenik IVh
Gama istrauje Boidar Bukili, uenik IIIl
Piramide u Gizi/Grobnice ili kosmiko savrenstvo 12 orovi Lilanda, uenik II10
Dakovi Jelena, uenik II10
Saznajte neto vie
Lakievi Milan, uenik IIIl
Nejednakost izmeu aritmetike i Mara Irena, uenik IVj
geometrijske sredine 16 Popovi Ivana, uenik IIIl
Odreivanja minimalne i maksimalne vrijednosti u Popovi Jovana, uenik I12
zadacima iz fizike primjenom nejednakosti Radevi Mihailo, uenik II10
Radoman Milena, uenik II2
aritmetike i geometrijske sredine 20
estovi Bojana, uenik IVh
Paradoksi i sofizmi 24 Tapukovi Matija, uenik II2
Zanimljiva matematika
Brojevi Lektor: Mirjana Perovi, prof.
Mirakuli 26 Priprema na raunaru i dizajn korice:
ibice 27 Rastko Pajkovi, uenik IIIi
Zanimljiva matematika 28 tampa: M Print - Podgorica
Zanimljiva fizika
Pitanja i odgovori 10+10 29
Logike igre
Sudoku 30
Zanimljiva matematika
Matematiko-logike zagonetke i problemi 32
Ajntajnov zadatak 33
Citati 34
Zanimljiva fizika 35
U razgovoru sa...
Intervju Beo Meruli 36
Odabrani zadaci
Matematika 37
Takmienja
Dravno takmienje 2008.
Matematika 38
Fizika 46
Informatika 57
U korak sa ...
Kako uiti informatiku 62
Odgovori i rjeenja 63

1
Poznati matematiari

Pitagora
ovjek koji je svuda vidio brojeve

R
oen je na ostrvu Samosu u VI v.p.n.e. (oko 580. Pitagora oenio. Pitagorejci su vjerovali u reinkarnaciju i
god.). Imao je 18 godina kada je uestvovao na propovijedali strogo vegeterijanstvo.
Olimpijskim igrama, gdje je odnio sve nagrade
u pesnienju. Poslije pobjede Pitagora se bavio muzikom
odluio je da putuje: Jonija, i matematikom. Kao filosof,
Sirija, Liban samo su neke od smatrao je da je sutina svih
destinacija, ali su za njegovu stvari broj i da se sve u vasioni
nauku najznaajniji susreti sa moe svesti na nepromjenljivu
mnogim uiteljima i misliocima harmoniju brojanih odnosa.
onog vremena, meu kojima Pripisuje mu se uenje o
je bio i Tales iz Mileta, kao i monadama (jedinice koje su isto
boravci u Vavilonu i Egiptu. to i besmrtne ljudske due),
U Vavilonu je u zarobljenitvu Pitagorina teorema, otkrie
izuavao tamonju matematiku, prve muzike skale i figure
dok je u Egiptu upoznao nazvane tetraktys (slika 1)
egipatsku geometriju i, na kraju, koja implicira dekadni sistem
postao egipatski svetenik, brojeva, zatim dokaz teoreme
upuen u njihove tajne obrede. da je zbir unutranjih uglova u
Kao pedesetogodinjak seli se u trouglu 180 stepeni, itd.
Kroton gdje osniva religiozno-
asketsko bratstvo (pitagorejce), Kako je za pitagorejce broj iskon
tajno drutvo posveeno svega, smatrali su da su jedini
prouavanju brojeva. dopustivi brojevi prirodni a
veliine one koje se mogu opisati
Moglo bi se rei da je Pitagora gotovo mitska linost kao njihovi odnosi (tj. kao racionalni broj). Ipak, zanimljivo
jer ak nema sauvanih prepisa njegovih originalnih je da je jedno od njihovih najveih otkria upravo suprotno
tekstova. Mogue da su i njegovi sljedbenici uitelju tom uvjerenju. Naime, radi se o nesamjerljivosti stranice i
osnivau pripisivali niz kasnijih sopstvenih matematikih dijagonale kvadrata (slika 2). Znamo da su dvije veliine,
postignua i filosofskih uvida. Kao kakav kaluerski ili a i b, samjerljive ako im je odnos racionalan broj. Danas,
viteki red, bratstvo je imalo svoja pravila i njegovalo pak, znamo da je odnos dijagonale d i stranice a kvadrata
strog nain ivota. Da bi izbjegli da njihove tajne saznaju jednak kvadratnom korijenu iz 2, koji je iracionalan
stranci koji nijesu pripadali zajednici, broj, pa je ta injenica oigledno u
veinom su svoja znanja prenosili Pitagora je prvi izrekao: suprotnosti s osnovnim vjerovanjem
usmeno. Napisano ostaje, usmeno Ja sam filosof pitagorejaca da je sve opisivo pomou
odlijee. Da njihove rijei ne bi
odletjele, usavrili su brojne vjebe za
nazivajui tako sebe onim prirodnih brojeva.
razvijanje pamenja. Stoga su postojali koji tei mudrosti. Oni su to, zapravo, i sami otkrili te je
akusmatici1, kojima su prenoeni to uzdrmalo temelje njihovog kulta.
rezultati, ali ne i dokazivanja neophodna da bi se do njih Matematiarima je uvedena zabrana iznoenja rezultata
dospjelo i matematiari2, kojima su prenoeni rezultati i van lanova unutranjeg kruga. Postoji anegdota da je
dokazi. Matematiare je poduavao sam Pitagora. enama Hipas prekrio tu zabranu i za kaznu bio prisiljen da izvri
je takoe bilo dozvoljeno da pripadaju redu. Meu njima samoubistvo skokom u more sa neke visoke litice.
najatraktivnija je bila mlada i lijepa Teano, kojom se
Postoje razliite pretpostavke o Pitagorinoj smrti. Neki
1 akuzmatikoi(gr.) - oni koji ive u svojim kuama, ak tvrde da je umro od sree kad je pronaao neku svoju
posjeivali su zajednicu samo tokom dana. teoremu. Zanimljivo, ali vjerovatno netano. Jedna od
2 mathematikoi(gr.) - ivjeli su stalno u zajednici, bez linih njih kae da je Pitagora, bjeei sa svojim uenicima
interesa

2
Poznati matematiari
iz Krotona, iji su se stanovnici bili digli protiv njega, - Pitagorejci su u matematiku uveli pojam savrenog
naiao na polje pasulja preko koga je trebalo umai broja, broja koji predstavlja zbir svih svojih pravih
progoniteljima. Poto su, prema vjerovanju, u pasulju djelilaca. Prvi savren broj je 6, zbir trojke, dvojke i
prebivale due pokojnika, Pitagora ne htjede da pregazi jedinice a estica je ujedno djeljiva svim ovim brojevima.
mahunarke. Radije u umrijeti nego da pregazim jadni Naredni savren broj je 28.
pasulj, rekao je i sjeo kraj polja. Stanovnici Krotona su
28 14
ga sustigli i masakrirali i njega i njegove uenike. Druga
6 3 28 7
teorija, pak, kae da je u miru proivio ostatak ivota...
6 2 28 4
1 6 1 28 2
+
6 28 1
+
28
- Pitagora je prvi podijelio brojeve na kategorije neparnih i
1 parnih, ali je otiao i korak dalje, personifikujui neparne
2 kao muke a parne kao enske.

- Pitagorejci su meu brojevima pronali brojeve


prijatelje dva broja koja imaju svojstvo da je jedan
Slika 1. figura tetraktys Slika 2. dijagonala jednak zbiru svih pravih djelilaca drugog i obrnuto.
- implicira dekadni sistem kvadrata stranice 1
brojeva Primjer: 220 i 284. Pravi djelioci broja 220 su: 1, 2, 4,
5, 10, 11, 20, 22, 44, 55 i 110, iji zbir daje 284. Pravi
Evo samo nekih od brojnih Pitagorinih otkria: djelioci broja 284 su: 1, 2, 4, 71 i 142, iji zbir daje
220. Anegdota kae, da je, kada su Pitagoru pitali kako
- uvena Pitagorina teorema iji je prvi pisani dokaz bi opisao pravog prijatelja, slavni matematiar i filosof
dao Pitagora, iako se danas zna da su je prije njega odgovorio: Prijatelj je kao drugo ja, kao broj 220 broju
koristili Egipani, Vavilonci, Indusi, Kinezi i druge stare 284
civilizacije. Ova teorema izazivala je veliko interesovanje
kroz istoriju matematike. Legenda kae da je, poto je 220 110
otkrio uvenu teoremu, Pitagora priredio hekatombu - 220 55
sveanost rtvovanja stotinu volova, tada zastupljenu u 284 142 220 44
Grkoj. Od tada, kau, volovi ne vole matematiku... 284 71 220 22
284 4 220 20
284 2 220 11
220 10
b2

284 1
+ 220 5
+

220
a2

c 220 4
=

220 2
c2

220 1
+
284
b
a - Smatra se da ime discipline - matematika svoje porijeklo
duguje pitagorejcima. Naime, ui krug njegovih
- Pitagora je otkrio da je muziki interval odnos dva broja, sljedbenika, koji su sebe nazivali mathematikoi, ime
pa je na primjer interval oktave 1/2, kvinte 2/3, a kvarte je oigledno dobio iz ritualnih i religijskih razloga.
3/4. Brojani odnosi tako postaju kadri da predstavljaju Pitagora im je, impresioniran ritualima egipatskih
muzike harmonije. Za pitagorejce, cijelo nebo je svetenika, dao ime po mathemi - nazivu za jednu
sazdano od muzikih ljestvica i brojeva, pa je muzika vrstu zemlje koju su egipatski svetenici koristili u
harmonija sfera. ritualima. Rije matematika, nastala na opisani nain, po
Pitagorinom miljenju, trebalo je da oznaava ono to
se ui.
Bojana estovi,
Gordana Boljevi
IV-h

3
Jedan problemvie rjeenja

Dokazi Pitagorine teoreme


Kvadrat nad hipotenuzom, to zna svako dete, jednak je zbiru kvadrata nad obe katete
Branislav Nui
U geometriji je veoma znaajna teorema koja nosi ime Pitagore, a odnosi se na pravougli trougao: zbir povrina kvadrata
konstruisanih nad katetama jednak je povrini kvadrata konstruisanog nad hipotenuzom. Drugim rijeima, ako su a i b
duine kateta i c duina hipotenuze onda je a + b = c
Prema onome to se zna, do danas nije poznato kako je i ko prvi dokazao ovu teoremu. Pretpostavlja se da je najprije
pronaen dokaz za jednakokrako pravougli trougao ABC, jer za ovakav trougao, povlaenjem dijagonala A1C, B1C, AB2
i BA2 neposredno proizlazi da se kvadrat ABB2A2 nad hipotenuzom sastoji iz etiri meusobno podudarna jednakokraka
trougla koja su podudarna sa jednakokrakim pravouglim trouglovima: A1AC , A1C2C, BB1C i B1C1C od kojih su
sastavljeni kvadrati nad katetama, kao i sa datim trouglom ABC, jer imaju jednake hipotenuze i uglove na njoj od 45.

C2 C1

C
A1 B1

A B B
A
A2 B2

Danas je poznato oko stotinu dokaza ove teoreme za proizvoljan pravougli C


trougao, ali se uoava da se dokazi mogu podijeliti u etiri grupe:
1. dokazi koji koriste razlaganje povrina (mozaiki dokazi).
2. dokazi koji koriste izjednaavanje povrina,
3. dokazi pomou raunanja,
4. dokazi koji koriste slinost geometrijskih figura.

Dokaz 1: papir i makaze


Teoremu moemo dokazati sa samo dva reza makazama. Ukoliko kvadrate ABCD i EFGH, izrezane od papira, postavimo
kao na slici dolje lijevo i spojimo take D i G sa B (ABF i BF = AB), nastae dva podudarna pravougla trougla. Pri
tom e manji kvadrat biti kvadrat nad manjom katetom, a vei nad veom. Sa dva reza makazama po BD i BG dobijaju
se djelovi od kojih se moe nasloiti kvadrat uz hipotenuzu bilo kojeg od dva podudarna trougla (slika dolje desno).
Povrina treeg kvadrata jednaka je zbiru dva data manja.

H G

D C

A BE B F

4
Jedan problemvie rjeenja

Dokaz 2
Jedan od dokaza potie od uvenog indijskog matematiara Bhaskare koji je roen 114. godine. Bhaskara je sastavio
etiri podudarna trougla ABC, EAH, DEF i BDG. Hipotenuza tih trouglova je jednaka stranici c, dok due katete
b tih trouglova obrazuju unutranji kvadrat HCGF duine stranica b a. Gledajui sliku, zakljuuje:

E D
c
G a
a b

ba F
(b a )
2
+ 4 = c 2 , odakle je a + b = c b
2 c b c
C
a
b
a
a b
H
c
A B
Dokaz 3: Garfildov dokaz
Dokaz potie od Dejmsa Abrahama Garfilda (1831-1881), koji je postao ameriki predsjednik 1881. godine.
Konstruiimo du BD normalno na AB, tako da je AB = BD = c i spustimo iz take D normalu DE na pravu koja sadri
stranu CB (raspored taaka na pravoj je C B E).
ACB i BED su pravougli trouglovi ija je stranica AB = BD = c i CBA = EDB (uglovi sa normalnim kracima),
odakle slijedi da su oni podudarni i da je ED = CB = a i AC = BE = b. etvorougao ACED je trapez.
C b A

a
c PACED = 2PACB + PABD
B
( a + b ) ( a + b ) = 2ab + c 2
(a 2
+ 2ab + b 2

) = ab + c 2

2 2 2 2 2
b c
a 2 + 2ab + b 2 = 2ab 2 2 2 2
, ime je dokaz zavren.
b+c a +b =c
E a D

Dokaz 4
D

Posmatrajmo pravougli trougao ABC sa pravim uglom kod tjemena C E


b Q
Konstruiimo kvadrat nad hipotenuzom c, tj. FBGA i kvadrate nad katetama a i b
2

b, odnosno ACDE i CBKQ Iz jednakosti stranica AC = AE, FA = AB i jednakosti C a


uglova EAB = CAF slijedi podudarnost trouglova EAB i CAF, tj. jednakost a 2 E
njihovih povrina, odnosno
b a K
b = x c (1). A cN B
Analogno uoavamo da su i trouglovi ABK i GBC podudarni (AB = BG,
CB = BK i ABK = GBC) odakle slijedi:
c xc yc c
a = y c (2).
Iz (1) i (2) slijedi:
a + b = (x + y) c = c, to je i trebalo dokazati. F x y G

5 C
C
Jedan problemvie rjeenja

Dokaz 5: vektorski
Pitagorinu teoremu moemo dokazati i koristei se svojstvima skalarnog proizvoda vektora.

Kako je vektor c jednak zbiru vektora a i b, to :
E
( )(
cc = a + b a + b )
2 2 2 2 2
c = a a + 2a b + b b = a + 2 a b cos 90 + b = a + b ,

odakle slijedi c = a + b, to je trebalo dokazati.


0 A

Dokaz 6: korienje Apolonijeve teoreme


c

Poznatu relaciju koja vlada meu stranicama pravouglih trouglova moemo dokazati i korienjem Apolonijeve
teoreme:
F
Ako je D taka stranice AB trougla ABC takva da je AD : BD = m : n, tada je

n AC + m BC = n AD + m BD + (m + n) CD C
Kod pravouglog trougla ABC, sa pravim uglom kod tjemena C, oigledno je
m = n = 1, pa prethodna relacija dobija oblik: a
b c
AC + BC = AD + BD + 2 CD, c c
A B
odakle slijedi: D
2 2 2
c c c
a 2 + b 2 = + + 2 , tj. a + b = c, ime je teorema dokazana.
2 2 2
A
Dokaz 7
Posmatrajmo pravougli trougao ABC sa uglovima , , koji odgovaraju stranicama duine a, b, c i neka je r
poluprenik krunice upisane u taj trougao. Neka su A1, B1 i C1 dodirne take krunice sa stranicama BC, CA i AB.
Obiljeimo sa P povrinu, sa s poluobim, a sa x, y i z respektivno duine parova tangentnih dui CB1 i CA1, AB1 i AC1,
BA1 i BC1 datog trougla C


Vai relacija: tg tg + tg tg + tg tg = 1, + + = , > 0, > 0, b > 0. a
2
2 2
2 2
2
Dokaz: b
A1 A B1 c
r r r
tg = , tg = i tg = , pa polazei od lijeve strane jednakosti dobijamo:
2 y 2 z 2 x

r r r r r r r ( x + y + z)
2

tg tg + tg tg + tg tg = + + = =
2 2 2 2 2 2 y z y x z x x yz

(r s)
2
r2 s r 2 s2 P2
= = = = 2 = 1.
x y z s x y z s ( s c) ( s a) ( s b) P
Vratimo se dokazu Pitagorine teoreme.

Kako je x = r i = 45
0
, dobijamo:
2

6
Jedan problemvie rjeenja

r2 (r + y + z) r (r + y + z) P
=1 =1 = 1 P = y z. C
r yz yz yz
Kako je P povrina pravouglog trougla, to je 2P = ab, pa je: x x=r
B1 A1
a b = 2 y z a b = 2 ( s a) ( s b)
b+ca a+cb r r=x
a b = 2
2 2 y z
b2 c2 a 2 O
a b = a b +
2 2 2 r
c2 = a 2 + b2
A B
y C1 z

Dokaz 8: Hofmanov dokaz


Ovaj dokaz (Majnic, 1821. god.) koristi vano svojstvo kruga:

Teorema: Ako je P taka van kruga k, u ravni tog kruga, tada je proizvod odsjeaka koje krug k odreuje na bilo kojoj
sjeici povuenoj iz take P jednak kvadratu odgovarajue tangentne dui.

Dokaz: Neka su B i C presjeci sjeice, povuene iz take P, sa krugom k. Tada imamo da je CPA~BPA
(CPA= BPA, CAP = ABP; ugao izmeu tetive i tangente CAP u jednoj krajnjoj taki tetive jednak je periferijskom
uglu ABP nad tetivom). Iz slinosti trouglova slijedi da je AP : PB = PC : AP AP = PC PB, to je trebalo dokazati.

C
P k

Koristei ovo svojstvo, dokazaemo Pitagorinu teoremu. Kada se oko tjemena A pravouglog trougla ABC, kao centra,
opie krug sa poluprenikom AC = b i hipotenuza AB produi do presjeka A1 sa ovim krugom, tada prema navedenoj
teoremi vai:

BC 2 = BA1 BB1 a 2 = ( c + b ) ( c b )
a 2 = c2 b2
c2 = a 2 + b2
C

b a

b
A1 A B1 c B

7
Jedan problemvie rjeenja

Dokaz 9: Simpsonov dokaz

Na kraju, dozvoliemo sebi da odstupimo od matematike strogoe zapisa dokaza teoreme i dati jednu interpretaciju
dokaza po Simpsonu (Elements de geometrie, Pariz, 1766.), koji se smatra moguim dokazom samog Pitagore. Dokaz
je na svoj nain izloio Leonard Mlodinov u svom djelu Euklidov prozor, koji ovdje navodimo u originalu:

Nikolaj Aleksej Nikolaj Aleksej

Nikolaj
za

za
nu

nu
Aleksej

Aleksej

Aleksej
j2
ote

ote

e
ks
Hip

Hip

le
A
Nikolaj
Aleksej
Hip Aleksej Hip
ote
Nikolaj

Nikolaj

Nikolaj
nuz ote

j2
a nuz

a
ol
a

ik
Aleksej Nikolaj NNikolaj Aleksej

Da bi se stvari pojednostavile, nazovimo nekako stranice trougla. Hipotanuza ve ima naziv, premda podui, pa
emo ga zadrati, ali emo staviti veliko poetno slovo, Hipotenuza, da bismo je razlikovali od matematikog naziva
hipotenuza. Neka se dve katete pravouglog trougla zovu Aleksej i Nikolaj. Igrom sluaja, upravo su ovo imena mojih
sinova. U asu dok ovo piem, Aleksej je vii od Nikolaja, pa neka vea kateta bude nazvana Aleksej, a manja Nikolaj
(dokaz ostaje na snazi i ako su katete jednake). Poeemo tako to emo nacrtati kvadrat ija je osnovica zbir Alekseja
i Nikolaja. Stavimo potom po jednu taku na svaku stranu kvadrata, na ono mesto gdje se susreu duina Alekseja
i Nikolaja, a potom spojimo te take. Dva naina koja nas zanimaju prikazana su na slici. Na prvom crteu imamo
kvadrat ija je osnovica Hipotenuza i etiri trougla vika. Na drugom crteu imamo dva kvadrata ije su osnovice
Aleksej i Nikolaj i dva pravougaonika vika koja se mogu dijagonalno prepoloviti, ime se dobijaju etiri trougla
istovetna onima koja predstavljaju viak na prvom crteu.
Preostaje samo raunanje. Dva izdeljena kvadrata imaju
istovetne povrine, tako da nam, kada uklonimo po etiri
ista trougla vika sa svakog od njih, preostaju dve
povrine koje moraju biti istovjetne. Jedna od njih je Mladi, siromani matematiar objanjava jednom
kvadrat ija je osnovca Hipotenuza, dok su u drugom
francuskom plemiu dokaz Pitagorine teoreme.
sluaju posredi dva kvadrata ije su osnovice Aleksej,
odnosno Nikolaj. Time je teorema dokazana! Objanjava strpljivo i polako, ali svaki put plemi
odgovara: Ne razumijem. Nakon vie uzaludnih
Literatura:
pokuaja mladi instruktor izgubi ivce: Monsenjor,
1. oko G. Markovi, Geometrijski poliformizam,
Podgorica, 2006. kunem vam se svojom au da je Pitagorina teorema
2. Borisav Simi, I to je matematika, Zavod za udbenike istinita! U taj tren plemi ustaje, ljubazno se nakloni,
i nastavna sredstva, Beograd, 1992. i s izrazom uenja kae: Trebali ste mi to odmah
3. Leonard Mlodinov, Euklidov prozor, Laguna, 2005. rei. Ne bi mi nikad palo na pamet da posumnjam u
4. http://alas.matf.bg.yu/~zlucic/view_doc.php?id=391 vau ast...
Bojana estovi
Gordana Boljevi
IV- h
8
Anegdote
o poznatim matematiarima

Euklid
Na kraju prvog predavanja koje je Euklid odrao jednoj grupi studenata poetnika,
A jedan od studenata upitao ga je: A ta e nam u ivotu matematika? Euklid nije
odgovorio nita. Nakon pola sata po svome robu poslao mu je jedan zlatnik i
D otpustio ga iz kole.
E
B Tales
Mlin
C Diogen Laertije pie: Hijeronim s Roda pria da je Tales, elei da pokae kako
je lako obogatiti se, predviajui da e biti dobar rod maslina, uzeo u zakup sve
mlinove maslina i tako nagomilao itavo bogatstvo. Istu su anegdotu prenijeli i
Ciceron i Aristotel, koji u Politici pie ovako:
Jer kad su mu zbog njegova siromatva prigovorili kako je filosofija
beskorisna, on je, kau, saznavi posmatrjui zvijezde da e te godine biti
dobar urod maslina, ve zimi s ono malo novca to je imao veoma povoljno
zakupio sve prese za ulje u Miletu i na Hiju, jer niko nije nudio vie novca. Kad
je zatim dolo pravo vrijeme, i nenadano su se i istovremeno traile mnoge
prese, iznajmljivao ih je po koliko je on htio, pa je, zaradivi mnogo novca,
pokazao kako je filosofima lako obogatiti se kad to hoe, ali to nije ono emu
oni tee. Govori se kako je na taj nain Tales dokazao svoju mudrost.

Gaus
Gaus se kao dijete isticao svojom otroumnou. Postoji anegdota da je
jednom prilikom iznenadio svoga uitelja koji je uenicima zadao da saberu
sve prirodne brojeve do 100. Samo to je uitelj sjeo, devetogodinji Gaus se
javio da je uradio zadatak.
Kako? pitao je uitelj.
Vrlo lako. Grupisao sam brojeve u 50 parova : u prvom su brojevi 1 i 100,
u drugom 2 i 99, u treem 3 i 98 itd. do pedesetog para u kome su brojevi 50
i 51. Zbir brojeva u svakom paru je 101, pa zbir brojeva od 1 do 100 iznosi
10150= 5050

Dirihle
Pria se da njemaki matematiar Dirihle nije bio mnogo razgovorljiv. Kada
mu se rodio sin,on je svojoj tati poslao telegram koji je vjerovatno najkrai u
istoriji telegrafije. Glasio je: 2+ 1 = 3
Bojana estovi
Gordana Boljevi
IV-h

9
Feljton
ISTORIJA MATEMATIKE - I DIO

Mesopotamija
Mesopotamija1, podruje izmeu i oko Eufrata i Tigra Tablice kvadrata i kubova, koje za broj n kazuju koliko
na Arabijskom poluostrvu, danas sredinji dio Iraka, je n i n. Pronaene su tablice jo iz 2000. godine prije
bila je kolijevka nekoliko najstarijih kultura. Govorei n.e. koje sadre kvadrate brojeva od 1 do 59 (to su sve
o matematici stare Mesopotamije, podrazumijevamo cifre u sistemu sa osnovom 60).
ostavtinu Sumera, Vavilonaca, Asiraca, Akaana, Tablice kvadratnih korijena pa, tavie, i kubnih korijena
Kaldejaca i drugih naroda koji su u razliitim razdobljima,
izraenih priblino pomou ezdesetina. Na primjer, 2
poev od treeg milenijuma prije nae ere (sumerski preiod)
pa sve do perioda od 3. v. pr. n.e. (period grke vladavine), su zapisivali kao 1 24 51 10 . Naravno u pitanju su
boravili na djelovima tog podruja. Izraz vavilonski odgovarajue oznake na klinastom pismu za odgovarajue
esto se koristi kao sinonim za mesopotamski. cifre u sistemu sa osnovom 60. Znai, broj je
24 51 10
Izvori informacija koji se odnose na nivo mesopotamske 1+ + + , to je priblino 1,414213... i razlikuje
matematike vrlo su obimni. Mnogo stotina tablica pisanih 60 602 603
klinastim pismom2 bave se problemima koje bismo danas se od 2 tek na sedmom decimalnom mjestu Smatra se
zvali algebarskim, ili se bave geometrijskim odnosima. da su poznavali uveni a i danas standardni metod za
Kako su raunali? izraunavanje kvadratnog korijena cifru po cifru.
U Mesopotamiji je razvijen brojevni sistem sa osnovom
60 Jo uvijek se ne moe sa sigurnou rei zato ba
osnova 60, ali je evidentno da ima nekoliko prednosti.
Da li znate zato se krug dijeli na 360?
Prvo: kod brojnih sistema sa veom osnovom , zapis broja Negdje oko 2400. pr. n.e., drevni Sumeri su primijetili
je krai. Druga, vanija prednost, je to broj 60 ima ak vidljivu krunu putanju Sunevog godinjeg puta
deset pravih djelioca 2,3,4,5,6,10,12,15,20 i 30, zbog preko neba i znali su da je potrebno oko 360 dana da
ega mnogo vie brojeva ima konanu reprezentaciju obavi svoj put. Prema tome, njima je bilo razumno da
nego u naem dekadnom sistemu. Trea prednost je stara podijele kruni put na 360 stepeni da bi mogli da prate
podjela godine na 360 dana (12 mjeseci po 30 dana). Tom dnevno kretanje Sunca. To je vodilo ka dananjem
broju dodavalo se jo 5 suvinih dana svake godine. krugu od 360 stepeni. Kako stepen dijelimo na 60
Vavilonci nijesu imali pogodne metode za vrenje raznih, minuta, a minute na 60 sekundi, to je i danas prisutan
pa i najprostijih aritmetikih operacija. Iz tog razloga u brojni sistem sa osnovom 60 koji su koristili stari
vavilonskoj matematici, caruju razne vrste tablica. Tako Vavilonci.
imamo, na primjer: Pokuavalo se ve dosta davno da se ova starovavi-
1 lonska podjela kruga zamijeni dekadnom, pri emu
Tablice recipronih brojeva i njihovih (tanih ili se krug dijelio na 400 jednakih djelova, gradi, svaki
m
grad na 10 decigradi, decigrad na 10 centrigradi, itd.
priblinih) vrijednosti u sistemu sa osnovom 60. Pomou Meutim, praktinije je ostati pri podjeli kruga na
njih su Vavilonci, izmeu ostalog, dijelili brojeve i to tako 360 jednostavno zato to broj 360 ima 22 inioca
1 koji obezbjeuju lake dijeljenje kruga.
to su ih svodili na mnoenje prema formuli ab = a .
b
Arheolozi su u posjedu tablica sa recipronim vrijednostima
brojeva sve do nekoliko milijardi; Tablice eksponencijalnih brojeva, tj. za izvjesne
Tablice mnoenja; vrijednosti osnove c, a za razne izloioce n koji se uzimaju
redom, daju se vrijedosti stepena cn. Na primjer, 2n za
n = 1, 2, 3, 4, itd. jednako je 2, 4, 8, 16, itd., a pri tome
1
Mesopotamija grki doslovno znai: (zemlja) izmeu nalazimo i osnove koje nijesu cijeli brojevi kao, npr. 4,5.
(dvije) rijeke
2
Sumeri su urezivanjem tragova na vlanim glinenim tablicama,
Dakle, jo jedna opta karakteristika stare mesopotamske
koje su se potom suile na suncu, stvorili prvo pismo. matematike jeste izrada tablica, kojom su se Vavilonci
Kako ti zarezi podsjeaju na crte klina, to je pismo sluili kao to se mi danas sluimo npr. logaritamskim
dobilo ime klinasto pismo. tablicama
10
Feljton

Vrhunac
drevne sumersko-vavilonske
aritmetike je, bez sumnje, rjea-
vanje algebarskih jednaina prvog,
drugog pa ak i treeg stepena, kao
i jednaina sa jednom i sa dvije
nepoznate. S obzirom da nijesu ra-
spolagali simbolima za nepoznate i poznate
brojeve, kao ni znacima raunskih radnji, njihova
algebra je bila retorika tj. sve se opisivalo rijeima.
Jedno od glavnih hendikepa vavilonske numerike Astronomija
matematike bilo je dugotrajno odsustvo nule, sve do
Generacije i generacije
3. vijeka pr. n.e. kada je uveden simbol za upranjeno
vavilonskih astronoma,
mjesto u zapisu broja ali je i danas njegova funkcija ostala
uglavnom iz redova svetenika,
nejasna
osmatrale su nebo i biljeile rezultate
Geometrija svojih osmatranja. Stotine, a prema nekim izvorima,
i hiljade godina osmatranja stvorile su ogromnu bazu
Geometrija Mesopotamije vjerovatno je ve oko 2000.
podataka. Uporedo sa time, samo ureenje posmatranja
godine prije n.e. raspolagala pravilima za izraunavanje
dalo je veliki doprinos matematici. Vavilonski astronomi
povrine pravougaonika, pravouglog i jednakokrakog
razvili su sferni koordinatni sistem, praktino isti kakav
(a moda i opteg) trougla kao i za obim pravouglog
je u upotrebi u savremenoj astronomiji. Osnovni krug
paralelopipeda i nekih posebnih pravih prizmi. Obim
nebeske sfere inila je prividna putanja Sunca tokom
kruga i obim krunog valjka raunalo se, u starije
godine. Kako je poloaj svakog nebeskog tijela odreen
vrijeme, s aproksimaciojom 3 za broj , a kasnije su
geografskom irinom i duinom u odnosu na taj krug,
Vavilonci upotrebljavali i mnogo bolju aproksimaciju
koordinate tijela tokom vremena su biljeene u itave
= 3 1/8 = 3,125 (greka samo oko 0,5%).
biblioteke glinenih tablica. Iako je biljeenje isto
Za Pitagorinu teoremu su znali i to u njenom optem aritmetiki posao, same tablice se mogu shvatiti, ako
obliku, ali najvjerovatnije ne i dokaz. Glinena tablica koja se poloaji zapisuju dovoljno dugo, kao prve tablice
datira iz perioda izmeu 1900. i 1600. god. pr. n.e. daje sinusoida. Razvoj trigonometrije i odgovarajuih
tabelu Pitagorinih trojki cijelih brojeva koji zadovoljavaju matematikih vjetina povezanih sa njom, u kasnijem
formulu: a + b = c. S pravom se pretpostavlja da je helenistikom periodu, skoro je u potpunosti pokrenut
sluila za rjeavanje pravouglog trougla. Ova tablica se zahvaljujui vavilonskim astronomskim osmatranjima.
u istoriji smatra prvim dokumentom iz teorije brojeva. Danas, kada na Grke gledamo kao na svoje intelektualne
U Britanskom muzeju se uva tablica koja sadri zadatak roditelje, treba napomenuti da je niz znaajnih grkih
koji se rjeava upotrebom Pitagorine teoreme. Tekst intelektualaca, meu kojima i sam Tales, svoja
glasi: osnovna matematika znanja donosio sa putovanja u
4 je duina a 5 je dijagonala. Kolika je irina? Nju ne Vavilon. Dakle, uticaj Vavilonjana sa svojim korpusom
znamo (ona je nepoznata). 4 puta 4 je 16. 5 puta 5 je 25. matematikih i astronomskih znanja i danas se prepoznaje
Ako oduzme 16 od 25, ostae 9. Koliko treba da uzme u naem intelektualnom naslijeu.
da bi ga pomnoivi dobio 9? 3 puta 3 je 9. 3 je irina. Milena Radoman, II-2
Vano je napomenuti da se ni u jednom vavilonskom Literatura:
dokumentu ne pominje dokaz kao matematiki pojam, 1. http://www.matf.bg.ac.yu/~zlucic/opstamat.pdf;
niti se moe naslutiti da su ikada osjetili potrebu da neto 2. http://ahyco.ffri.hr/seminari2007/povijestmatematike.htm;
dokau. Matematiku su, uglavnom, shvatali kao praktinu 3 Pregled istorije i filozofije matematike, Milan Boi, Zavod
vjetinu. za udbenike i nastavna sredstva, Beograd, 2002.

11
Gama istrauje

Piramida u Gizi
grobnica ili kosmiko savrenstvo
Znate li da nabrojite svjetska uda starog svijeta? Jedno meu njima je posebno.
To je jedino ouvano drevno svjetsko udo. Da, pogodili ste.
To je Keopsova piramida.

Matematika dimenzija piramide


Istorija i nastanak
Nemojte da vas ovaj naslov navede da odustanete prije nego
proitate tekst. Neemo samo navesti dimenzije, uporediemo Uili smo da su piramide stvorene oko 2560.
ih sa odnosima u matematici, a onda ete proviriti u tajne ove g.p.n.e. Pretpostavlja se da su ih gradili stari
udesne graevine. Biete iznenaeni, ali spremni za nastavak... Egipani i da su sluile kao grobnice egipatskim
faraonima. Da li ste kada posumnjali u to?
Nauka ni danas ne moe da odgovori na pitanje kako su te drevne Sigurno jeste, ba kao i veliki broj uenih ljudi
civilizacije znale za geometrijske odnose kasnije nazvane , , koji smatraju da ne moemo sigurno rei ni ko
Pitagorina teorema. Sve te odnose vidimo skladno ugraene u ih je sagradio ni kad su sagraene. O njihovom
dimenzijama piramide. Ono to zadivljuje su mjere koje pokazuju nastanku stvorene su legende od one da ih je
nevjerovatnu preciznost. izgradila civilizacija koja je postojala prije nae,
pa do toga da su ih gradili vanzemaljci. Kad
Iako (pretpostavimo) nijesu poznavali Pitagorin trougao, govorimo o piramidama, nita nije nemogue.
upravo takav trugao ini polovinu povri koji se dobija ako se Zato se neete iznenaditi kad kaemo da su
piramida presijee vertikalnom ravni koja prolazi kroz apoteme to najsavrenije graevine na svijetu, ali i to
naspramnih strana. Stranice dobijenog trougla stoje u razmjeri da je onaj ko ih je gradio obratio panju na
3 : 4 : 5! matematiku u svakom dijelu piramide. Odlino
je uklopio geometrijske odnose. Zato ona traje
Odnos obima osnove i dvostruke visine odgovara odnosu do danas, i trajae jo dugo...
obima kruga i njegovog prenika, odnosno broju .

Apotema i polovina osnove su u odnosu zlatnog presjeka, tj. njihov odnos je jednak broju .

Ugao izmeu bone strane i osnove priblino je jednak broju sedmica u godini, odnosno nagibu fosfatne
kiseline u DNK 52.

Stranice Velike piramide u perfektnoj su liniji sa stranama svijeta. Preciznost je zadivljujua


5 4 99,99%. Piramida predstavlja najpreciznije orijentisanu graevinu na svijetu, preciznije
orijentisanu i od Meridijanske zgrade u sklopu Grininog opservatorijuma.

3 Interesantno je da duina strane kvadratne osnove daje 365,342 egipatskih lakata. Broj
je identian broju dana tropske sunane godine!

Dimenzije piramide i planeta Zemlja


Moda ste negdje uli ili proitali o dimenzijama nae palnete skladno ugraenim u Veliku piramidu. Ako nijeste, imate
priliku da saznate koliko su graditelji ovog drevnog uda bili svjesni dimenzija planete Zemlje:

Trebalo je 22 vijeka da se izrauna udaljenost izmeu Zemlje i Sunca (149 400 000 km). To bismo, priblino, znali
mnogo ranije da se neko sjetio da sa 1 000 000 000 pomnoi visinu Kepsove piramide u Gizi.

Zapremina piramide pomnoena sa vrijednou gustine blokova od kojih je izgraena daje srednju vrijednost gustine
Zemljine kugle!

12
Gama istrauje

geografska irina
AZIJA
EVROPA
SJEVERNA
AMERIKA
geografska duina Velike piramide
AFRIKA

Velike piramide
JUNA
AMERIKA
AUSTRALIJA

Geodetska lokacija Velike piramide u Gizi na 30 stepeni sjeverne


geografske irine (jedna treina puta izmeu ekvatora i Sjevernog
pola) i u centru svjetske kopnene mase pogodne za ivot.

Odnos zapremina piramide i nae planete je tano 1:1015!

Egipatski lakat bio je podijeljen na 25 palaca, a svaki palac ima 25.4265 milimetara. Pomnoimo li to sa 100 milijardi,
dobijamo priblino tanu vrijednost putanje koju u svom okretanju oko Sunca proe Zemlja u jednom danu!

Obim osnove Velike piramide i njena visina stoje prema ekvatorijalnom obimu Zemlje i njenim polarnim radijusom
u razmjeri 1:43 200. Prema tome, Velika piramida bi mogla da bude precizno umanjen model sjeverne Zemljine
hemisfere! I ne samo to: specijalni broj iz razmjere izveden je iz jednog od kljunih planetarnih kretanja Zemlje,
tj. stope njene aksijalne precesije1

Mjerenja novijeg vremena pokazala su da je Keopsova piramida smjetena tano u centar kopnene mase Zemlje, na
meridijan koji presijeca najvie kopna i najmanje mora. Uzevi u obzir ukupnu povrinu svih kontinenata i povrinu
koju zauzima piramida, te stoga i potencijalno moguih 3 milijarde mjesta na kojima je mogla biti izgraena, anse
za tu sluajnost su 1: 3 000 000 000!

Kraljeva odaja i sarkofag u matematikim odnosima


Da ipak pretpostavimo da su piramide graene kao grobnice i zavirimo u kraljevu odaju. Tamo emo naii na nevjerovatne
geometrijske odnose

Kraljeva odaja formira savren pravougaonik koji izraava i pokazuje primjer zlatnog presjeka. Interesantno je i da
je smjetena tano na sredini vertikalnog presjeka piramide, u sloj ija je povrina jednaka polovini povrine osnove
piramide, gdje je dijagonala jednaka duini osnove i gdje je irina fasade jednaka polovini dijagonale osnove. Visina
kraljeve odaje je tano polovina dijagonale njenog poda! Nakon svega ovoga nee vas ni iznenaditi injenica da masa

1 precesija - kretanje koje izvodi rotirajue tijelo zbog djelovanja sile koja nastoji da promijeni smjer osovine rotiranja
tijela u prostoru. Zbog precesijskog kretanja Zemlje, njena osovina opisuje jednom u 26 000 godina potpuni
omota kupe, pri emu nebeski pol opie krunicu meu zvijezdama. Zemljina precesija nastaje djelovanjem
Sunca i Mjeseca na Zemlju, koja nije pravilna kugla, ve geoid.
13
Gama istrauje

Zlatni presjek Keopsove piramide sveden u razmjere:


pola osnove: visina = dijatesaron-kvarta
u laktovima: 220 : 280 = 0,786
u stopama: 380 : 482 = 0,788 }
= 0,75

pola osnove: apotema = dijapenta-kvinta


u laktovima: 220 : 356 = 0,618
u stopama: 380 : 613,78 = 0,618
= }

ap
pola osnove: cijela osnova = dijapazon-oktava

ote 13.7
ma 8 s
u laktovima: 220 : 440
}

6
: 3 top
= 0,5

56
u stopama: 380 : 760

lak
ata
a


osn 80 st
ov
280 lakata
492 stope

e : pa
22
0l
o
kraljeva dvorana

ak
ata
venti-
lacija
ventilacija velika dvorana
prolaz ulaz
kraljiina dvorana nagore

stijena
lana grobnica prolaz nadolje

220 lakata
380 stopa
440 lakata
670 stopa

Keopsovog sarkofaga iznosi tano 1015 puta manje od mase Zemlje! Dodajmo i to da je kraljeva odaja smjetena na
treini visine piramide, a prisjetimo se da je Keopsova piramida smjetena skoro tano na treini puta izmeu Sjevern-
nog pola i Ekvatora.

Piramida i astronomija
Pitate se kako je mogua veza ove vrste? Piramide ne prestaju da iznenauju:
Djelo Roberta Bovala The Orion Mystery detaljno obrazlae da raspored tri
piramide u Gizi odslikava poziciju tri glavne zvijezde Orionovog sazvjea.
Pravo iznenaenje otkriveno Bovalovim astronomskim proraunom bilo je
sljedee: Uprkos injenici da neki aspekti Velike piramide astronomski imaju
veze s dobom piramida, spomenici u Gizi su, kao cjelina, rasporeeni tako da
daju sliku neba (iji se izgled tokom vijekova mijenja), ali ne onakvu kakva je
bila 2500. godine stare ere, u razdoblju etvrte dinastije, ve onakvu kakva je bila,
samo tada, negdje oko 10450. godine stare ere! Da li nam to govori da su piramide
skoro 8 000 godina starije nego to mislimo i da je 10450. g.p.n.e. postojala veoma
napredna civilizacija za koju dananja istoriografija ne zna? ta kaete na ovo:
oko 10450 g.p.n.e. poela je era Lava, a svi znamo kako izgleda Sfinga!

14
Gama istrauje
Keopsova piramida i molekul DNK
Znamo ta je DNK: molekul koji predstavlja ivot svakog
od nas. Piramida se moe dovesti u vezu i sa ovim
izvorom ivota:
Prosjeni ugao pod kojim se vezuju azotaste baze bio bi
5145 , a kod Keopsove piramide ugao nagiba bonih
strana je 5151!
Odnos duine veze citozin-guanin (1,08 nanometara)
prema duini timin-adenin (1,11 nanometara), jednak
je 0,973, a bona ivica Keopsove piramide odnosi se
prema duini strane osnove kao 0,946!
Odnos prenika molekula DNK (2 nanometara) prema
duini koraka spirale (3,4 nanometara) bio bi 0,588. Ako
apotemu bone strane Keopsove piramide podijelimo
dijagonalom osnove dobijamo 0,568.
Ugao uspona spirale DNK iznosi 26, a ugao nagiba
glavne galerije na Keopsovoj piramidi - takoe 26!
U korak spirale DNK (34 angstrema) smjeteno je 10
nukleotidnih parova, ije je meusobno rastojanje 3,4
angstrema ili 1/10 koraka. Keopsova piramida ni tu ne
zastaje! Njena visina je oko 148m, a ulaz u piramidu
nalazi se na visini od 14m - priblino 1/10.
Ovo je samo djeli tajne neprevazienih graevina, naizgled jednostavnih, ali u stvari neistraenih i punih misterija.
Ukazali smo na veze piramide s naim ivotom, naom planetom. Zato momo rei da piramida sadri proporcije
Sunevog sistema kao makrokosmosa i brojeve ovjeka kao mikrokosmosa. Piramida je riznica proporcija, brojki,
tajni. ovjek se plai vremena a, kako kae stara arapska izreka, Vrijeme se plai piramida. Ne znam da li iko moe
biti ravnoduan i ne zapitati se ta su, u stvari, piramide, ko ih je stvorio, kako i zato je taj neko tako dobro poznavao
geometriju i odnose u njoj koji su otkriveni hiljadama godina poslije njihovog nastanka...
Milan Lakievi
III-l
Literatura:
1. Dragan Vianovi, Tajne piramida, PRJ Zemun, 1994.
2. Grejem Henkok, Tragovi bogova, Mono & Manana, Mo knjige, 2004.
3. Grejem Henkok, Robert Boval, uvari ovjeanstva, Mono & Manana, Mo knjige, Beograd 2005.
4. er Doci, Mo proporcija, Stylos, Novi Sad, 2005.

L
a
M
k
i
i
l
a e
n v
i Hieroglife napisala
Vesna Kovaevi
profesor

15
Saznajmo neto vie

Nejednakost izmeu aritmetike i


geometrijske sredine
Nejednakost izmeu aritmetike i geometrijske sredine (u daljem tekstu AG nejednakost) jedna je od najpoznatijih
algebarskih nejednakosti. S obzirom na to da srednjokolci, uglavnom, nijesu u prilici da se na redovnoj nastavi
matematike upoznaju sa ovom nejednakou, evo prilike da se kroz asopis upoznate sa ovim veoma praktinim
alatom.

Definicija aritmetike i geometrijske sredine


Neka su a1, a2, ... an proizvoljni pozitivni realni brojevi i nN
a1 + a2 + ... + an
Aritmetika sredina brojeva a1, a2, ... an definisana je izrazom .
n
Geometrijska sredina brojeva a1, a2, ..., an definisana je izrazom n a1a2 ...an .

Nejednakost
Vezu izmeu aritmetike i geometrijske sredine daje sljedea teorema:
Neka su a1, a2, ..., an proizvoljni pozitivni realni brojevi. Tada vai da je:
a1 + a2 + ... + an n
a1a2 ...an , n .
n

Jednakost se postie ako i samo ako je a1= a2= = an


Zavirimo li u istoriju, saznaemo da su za AG nejednakost znali jo i pitagorejci, ali se njen prvi dokaz (za n = 2)
pripisuje Euklidu.

Dokazi AG nejednakosti
Danas se u literaturi mogu nai desetine razliitih dokaza ove nejednakosti. Opti sluaj se dokazuje pomou matematike
indukcije, a mi emo ovdje razmotriti tri dokaza teoreme za n = 2, i to: algebarski, jedan geometrijski i jedan analitiki
a+b
dokaz, tj. razmatraemo dokaz nejednakosti ab , koja vai za svaka dva pozitivna realna broja a i b. Jednakost
2
se dostie ako i samo ako je a = b

Algebarski dokaz:
a+b
( )
2
a b 0 a 2 ab + b 0 a + b 2 ab ab ,
2
pri emu su a i b pozitivni realni brojevi. Jednakost se dostie ako i samo ako je a = b b a
a
Geometrijski dokaz:
b
Vei kvadrat ima stranicu duine a + b i njegova je povrina oigledno vea od povrine (b a)
etiri pravougaonika sa stranicama a i b. Dakle, imamo: b
a+b
( a + b ) 4ab a + b 2 ab a
2
ab , to je trebalo dokazati.
2
a b

16
Saznajmo neto vie
Jednakost se postie ako i samo ako je povrina velikog kvadrata jednaka povrini etiri pravougaonika, odnosno ako i
samo ako kvadrat u sredini figure iezava, a to se dogaa ako i samo ako je b a = 0, tj. a = b

Analitiki dokaz:
Funkcija f ( x) = e x je konveksna, to geometrijski znai da je grafik funkcije izmeu dvije y = ex
take na grafiku uvijek ispod tetive koja spaja te dvije take. Na grafiku eksponencijalne b
funkcije izaberimo dvije take sa koordinatama ( x1 , e x1 ) i ( x2 , e x2 ) i uvedimo oznake
a = e x1 i b = e x2 .
Prava kroz izabrane take ima jednainu: a+b
2
ba
y b = ( x x2 ) ab
x2 x1
x1 + x2 a+b a
pa taka na pravoj sa apsicom ima ordinatu .
2 2
S druge strane taka sa istom apscisom, ali na grafiku eksponencijalne funkcije ima
ordinatu
x1 + x2
x1 x1+ x2 x2
1 1
e 2
= ( e x1 e x2 ) 2 = ( a b ) 2 = ab
2
a+b
Kako se taka na grafiku funkcije nalazi ispod take na tetivi, imamo da je ab , tj.
2
slijedi AG nejednakost

Primjene AG nejednakosti
AG nejednakost je snano sredstvo za dokazivanje raznih sloenijih nejednakosti kao i zadataka drugog tipa pa se u
literaturi mogu nai brojni zadaci koji se svode na njenu primjenu. Evo nekoliko primjera.
a2 b2
1. Dokazati da je za svako a > 1 i b > 1; + 8
b 1 a 1
Rjeenje:

Na osnovu AG nejednakosti imamo da je:

a2 b2 a2 b2 ab a b
+ 2 =2 =2 (1)
b 1 a 1 b 1 a 1 a 1 b 1 a 1 b 1
x
Dokaimo jo da je 2 , za x > 1
x 1
x
( x 2) 0; x 2 4 x 4; x 2 4 ( x 1) ;
2
2
x 1
a2 b2
Ako u (1) primijenimo dokazanu nejednakost, dobijamo: + 2 2 2 = 8, ime je dokaz zavren.
b 1 a 1
1 1
2. Neka je n N Dokazati da je 1 + + ... + + n > n n n + 1.
2 n
Rjeenje:

1 1 1 1 3 n + 1 AG 3 n +1
1+ + ... + + n = (1 + 1) + + 1 + ... + + 1 = 2 + + ... + > n n 2 ... = n n n +1
2 n 2 n 2 n 2 n

17
Saznajmo neto vie
1
3. U jednaini x 2 + px = 0, p R \ {0}, vai x14 + x2 4 2 + 2 gdje su x1 i x2 rjeenja jednaine Dokazati
2 p2
Rjeenje:
1
Na osnovu Vietovih pravila imamo da je x1 + x2 = p i x1 x2 =
2 p2
2

(( x1 + x2 ) ) 1 1
2
( )
2 2
x14 + x2 4 = x12 + x2 2 2 x12 x2 2 = 2 x1 x2 2 x12 x2 2 = p 2 + 2 2 4 =
p 4p
1 1 1 AG 1
= p4 + 2 += 2 + 2 p4 + 4
2 + 2p 4 = 2 + 2
2 p4 2 p4 p
xy yz xz
4. Nai najmanju vrijednost izraza s = + + ako su x, y i z pozitivni brojevi ija je suma kvadrata jednaka 1.
z x y
Rjeenje:
x2 y 2 y 2 z 2 x2 z 2
s2 =
z2
+ 2 + 2 + 2 x2 + y 2 + z 2 =
x y
( )
1 x2 y 2 x2 z 2 1 x2 z 2 y 2 z 2 1 y 2 z 2 x2 y 2 AG
= + 2 + 2 + 2 + 2 + 2 +2
2 z2 y 2 y x 2 x z
x2 y 2 x2 z 2 x2 z 2 y 2 z 2 y 2 z 2 x2 y 2
2 + 2 + 2 + 2 = x2 + y 2 + z 2 + 2 = 3
z2 y y2 x x2 z

Kako smo dobili da je s 2 3 , slijedi da je s 3 tj. najmanja vrijednost datog izraza je 3.


3
Jednakost se dostie za x = y = z = .
3

2 2
5. Rijeiti sistem jednaina 2 x +y
+ 2 x + y = 8, x + y = 2.

Rjeenje:
2 2 2
+ x + y2 + y
Na osnovu AG nejednakosti imamo 2 x +y
+ 2x+ y 2 2x pa iz prve jednakosti sistema dobijamo
2 2
x +x+ y + y4 (1)

( )
2
Kako je 2 ( x + y ) x + y , iz druge jednakosti sistema slijedi

x+ y2 (2)
Iz 2 ( x 2 + y 2 ) ( x + y ) i (2) slijedi
2

x2 + y 2 2 (3)
2 2
Iz (1), (2) i (3) slijedi x + y = 2 i x + y = 2 , odakle rjeenjem sistema jedne linearne i jedne kvadratne jednaine
dobijamo x = y = 1.

6. Ako za stranice trougla ABC vai ( a + b + c ) abc = 16 P , gdje je P njegova povrina, dokazati da je trougao
2

jednakostranian.

Rjeenje:
Posmatrajmo Heronov obrazac, P = s ( s a ) ( s b ) ( s c ) , gdje je s poluobim trougla.

Neka je x = s a, y = s b, z = s c, ( x, y , z > 0 ) .

18
Saznajmo neto vie

Tada je s = x + y + z , P = ( x + y + z ) xyz ,
pa na osnovu uslova zadatka vai:
2 ( x + y + z ) ( y + z ) ( z + x ) ( x + y ) = 16 ( x + y + z ) xyz , tj

( x + y ) ( y + z ) ( x + z ) = 8 xyz (1)
Na osnovu AG nejednakosti vai:
x + y 2 xy , y + z 2 yz , z + x 2 zx (2)
odakle slijedi da je ( x + y ) ( y + z ) ( z + x ) 8 xyz. Na osnovu (1) mora biti x + y = 2 xy , y + z = 2 yz i

z + x = 2 zx . U (2) jednakosti vae za x = y, y = z i x = z Dakle x = y = z s a = s b = s c a = b = c.


1 1
7. Niz (xn) zadovoljava relacije 0 < xn < 1, xn +1 (1 xn ) za svako n Dokazati da je lim xn = .
4 n 2
Rjeenje:
AG 1
Iz 1 = (1 xn ) + xn 2 (1 xn ) xn i (1 xn ) xn +1 2 (1 xn ) xn +1 1 slijedi da je xn < xn +1 pa je niz (xn)
4
2
1 1
monotono rastui. Kako je niz i ogranien, to on konvergira. Neka je lim xn = x. Tada je x (1 x ) x 0,
n 4 2
1 1
pa je x = , tj. lim xn = .
2 n 2
A sada ostavljam itaocu da primjenom AG nejednakosti rijei zadatke koji slijede. Rjeenja zadataka bie objavljena
u sljedeem broju asopisa
Mirjana Pjei

Zadaci:
1. Brojevi a, b, c i d su pozitivni i njihov proizvod je jednak 1. Dokazati da je:
a 2 + b 2 + c 2 + d 2 + ab + ac + ad + bc + bd + cd 10.
2. Dokazati da je log 4 5 + log 5 6 + log 6 7 + log 7 8 > 4, 4.
3. Neka su a i b duine kateta, a c duina hipotenuze pravouglog trougla i i veliine otrih
2ab
uglova tog trougla. Dokazati nejednakost cos 2 2 .
2 c
4. Dokazati da ako su i otri uglovi i tg = 3tg , tada je 30 + .
5. Duina najvee stranice jednakokrakog trapeza je 13, a obim 28. Nai stranice trapeza ako je
njegova povrina 27.

Literatura:
1. http://e.math.hr/agnejednakost/index.html
2. Dr R. epanovi, dr M. Ani, Zbirka rijeenih zadataka iz matematike za takmienja uenika srednjih kola,
Laboratorija za matematiku i raunarstvo, Podgorica, 2005.
3. V. Dragovi, P. Mladenovi, S. Ognjanovi, Pripremni zadaci za matematika takmienja za uenike srednjih kola,
Drutvo matematiara, Beograd, 1999.
4. M. Obradovi, D. Georgijevi, Matematiskop 4, Odabrani zadaci za drugi razred srednjih kola, Matematiskop,
Beograd, 1995.

19
Saznajmo neto vie

Odreivanja minimalne i maksimalne vrijednosti


u zadacima iz fizike primjenom nejednakosti
aritmetike i geometrijske sredine
Pojava infinitezimalnog rauna predstavlja prelomni trenutak u istoriji ovjekovih stremljenja ka upoznavanju novog:
njegovim stvaranjem ljudi su dobili moan aparat za analizu najraznovrsnijih procesa, za dublje objanjavanje fizikih
pojava i formiranje naunog pogleda na svijet. Pri tome, primjena infinitezimalnog rauna u kojoj se on uenicima i
studentima koji osjeaju matematiku najjae predstavlja svojom jednostavnou, ljepotom i loginou, svakako je
odreivanje maksimuma i minimuma.

Mogunosti odreivanja maksimuma i minimuma u srednjoj koli


Odreivanje maksimuma i minimuma esto je veoma vano u nastavi prirodno-matematike grupe predmeta u srednjoj
koli. Meutim, elementi infinitezimalnog rauna, i prema prolim i prema reformisanom nastavnom planu i programu
opte gimnazije, izuavaju se tek u etvrtom razredu.
U ovom radu izlae se jedan od elementarnih naina odreivanja maksimuma i minimuma bez primjene infinitezimalnog
rauna. On, naravno, ima znatno uu primjenljivost u odnosu na infinitezimalni raun.

Nejednakost aritmetike i geometrijske sredine


Nejednakost aritmetike i geometrijske sredine N pozitivnih brojeva x1, x2,..., xn ima oblik:
x1 + x2 + ... + xn n
x1 x2 xn , (*)
n
pri emu znak jednakosti vai ako i samo ako je x1 = x2 = = xn
Dokaz ove nejednakosti moe da se nae u gotovo svim standardnim udbenicima matematike analize. Kao ilustraciju
njene direktne primjene u fizici uzimamo sljedei primjer:

Primjer 1. Odrediti minimalnu vrijednost odnosa ekvivalentnog otpora N redno vezanih otpornika razliitih otpora i
ekvivalentnog otpora njihove paralelne veze.

Rjeenje:
Ekvivalentni otpor redno vezanih otpornika iji su otpori r1, r2, ..., rn je R1 = r1 + r2 + ... + rn,
1 1 1 1
a ekvivalentni otpor njihove paralelne veze R2 moe da se odredi iz jednaine = + + ... + .
R2 r1 r2 rn
Traeni odnos je
R1 1 1 1
= = ( r1 + r2 + ... + rn ) + + ... + .
R2 r r
1 2 rn

Poto iz (*) imamo da je


1 1 1
r1 + r2 + + rn n n + + ... + i
r1 r2 rn

1 1 1 1 1 1
+ + ... + n n ,
r1 r2 rn r1 r2 rn
onda zakljuujemo da je n 2
Prema tome, minimalna vrijednost traenog odnosa je min= n 2

20
Saznajmo neto vie
Posljedice nejednakosti i njena primjena
Iz nejednakosti (*) imamo:
1. Najmanja vrijednost zbira n pozitivnih brojeva x1, x2, ..., xn je
( x1 + x2 + ... + xn )MIN =n n x1 x2 ... xn (1)
za x1 = x2 = = xn
2. Najvea vrijednost proizvoda n pozitivnih brojeva x1, x2, ..., xn je
n
x + x + ... + xn
( x1 x2 xn )MAX = 1 2 (2)
n
za x1 = x2 = = xn
Ako je n = 2, onda jednakosti (1) i (2), uzimajui da je x1 = A i x2 = B, glase:
( A + B )MIN = 2 A B (3)
2
A+ B
( A B )MAX = (4)
2
za A = B
Jednakosti (3) i (4) lako se dokazuju. Poimo od tvrenja

( )
2
A B 0, (5)

pri emu znak jednakosti vai u sluaju kada je A = B. Iz (5), kvadriranjem, slijedi
A 2 A B + B 0 , odnosno:
A + B 2 A B (6)
Daljim transformacijama se dobija:
2
A+ B
A B (7)
2
Nejednakosti (6) i (7) ine dokaz zavrenim.
Dakle, maksimum ili minimum moemo odrediti pomou neke od jednakosti: (1), (2), (3) ili (4).

Primjer 2. Otvoreni cilindrini sud visine H, do vrha je napunjen vodom. Na kojoj visini h od dna suda treba napraviti
otvor da bi domet mlaza d bio maksimalan? (Povrinu otvora smatrati zanemarljivo malom u odnosu na
povrinu poprenog presjeka suda.)
g
Rjeenje:
v
Prema Torielijevoj teoremi mlaz na izlazu ima brzinu
v = 2g ( H h) H h
Ova brzina ima horizontalni pravac, tako da se dalje
kretanje mlaza moe posmatrati na sljedei nain:
d
- kako du horizontalnog pravca ne djeluje nikakva
sila, mlaz u tom pravcu prelazi put d = vt ;
gt 2
- kako mlaz du vertikalnog pravca slobodno pada, to je preeni put za isto vrijeme h =
2
Eliminisanjem vremena t iz ovih jednaina dobijamo da je d = 2 h ( H h )
Maksimalni domet, u oznaci dMAX , mlaz dostie za odgovarajuu maksimalnu vrijednost izraza h(H h).
2
H H
Prema (4) imamo da je h ( H h ) MAX = , za h =
2 2
H
Slijedi: maksimalni domet mlaza je dMAX= H, za h =
2
21
Saznajmo neto vie
Primjer 3. Na horizontalnoj podlozi nalaze se tri lopte masa m1, m2 i m3, respektivno. Centri lopti nalaze se na jednoj
pravoj. Prvoj lopti saoptena je brzina v prema drugoj lopti. Poslije toga dolazi do apsolutno elastinih
centralnih sudara lopti. Kolika treba da bude masa druge lopte (m2) da bi, poslije sudara sa treom loptom,
brzina tree lopte bila maksimalna?

Rjeenje:
Iz zakona odranja impulsa i zakona odranja energije za centralni apsolutno elastini sudar prve i druge
lopte:
m v2 m v2 m v2
m1v = m1v1 + m2v2 i 1 = 1 1 + 1 2
2 2 2
dobija se da je brzina druge lopte poslije sudara
2m1v
v2 = ,
m1 + m2
pri emu je v1 brzina prve lopte poslije sudara sa drugom loptom.
Za centralni apsolutno elastini sudar druge i tree lopte zakon odranja impulsa i zakon odranja energije
moemo zapisati u obliku:
m2v22 m2u22 m3v32
m2v2 = m2u2 + m3v3 i = + ,
2 2 2
2m2v2
gdje su v2 i v3 brzine druge i tree lopte poslije njihovog sudara. Iz ovih jednaina je v3 =
m2 + m3
Uvrtavanjem izraza za brzinu v2 imamo da je
m1m2
v3 = 4v = 4v f (m1 , m2 , m3 )
(m1 + m2 )(m2 + m3 )
m1 m2
Uvodei oznake =A i = B,
m1 + m2 m2 + m3
imamo da je f (m1 , m2 , m3 ) = A B
Prema jednaini (4), brzina v3 e imati maksimalnu vrijednost za A = B, tj.
m2 = m1 m3
To znai da e lopta imati najveu brzinu ako je vrijednost mase druge lopte geometrijska sredina vrijednosti
masa prve i tree lopte.

q
Minimalna vrijednost funkcije y ( x) = px + i primjeri primjene
x
Posmatrajmo funkciju
q
y ( x) = px + , x > 0.
x y
q
Uvoenjem oznaka px = A i = B krae imamo da je y = A+B
x
Na osnovu jednaine (3) je
ymin
q
yMIN = 2 AB = 2 pq za A = B, tj. x0 = 0 x0 x
p
Razmotrimo sada dva primjera primjene ovog zakljuka.

22
Saznajmo neto vie
Primjer 4. Odrediti minimalnu vrijednost poetne brzine takastog tijela da bi bez promjene smjera kretanja prelo put
s0 = 100m. Taka se kree ravnomjerno usporeno stalnim ubrzanjem a = 2 m/s 2

Rjeenje:
at 2 a x
Poloaj take odreen je jednainom x = v0t , odakle je v0 = t +
2 2 t
q
Na osnovu prethodno izloenog odreivanja minimuma funkcije y ( x) = px + moemo uzeti da je
x
at x
A= i B=
2 t
Tada je
v0MIN = 2 AB = 2as0 = 20 m /s
Primjer 5. Na raspolaganju imamo 24 akumulatora elektromotorne sile E = 2V i unutranjeg otpora r = 0,3.
Akumulatore veemo redno (paralelno) u nekoliko jednakih segmenata, a zatim te segmente poveemo
paralelno (redno). Kolika se maksimalna jaina struje moe dobiti u spoljanjem kolu ako na tako vezane
akumulatore prikljuimo otpornik otpora R = 0,2?

Rjeenje:
Neka je N ukupan broj akumulatora, a n broj akumulatora u jednom segmentu Tada je s = N/n broj
segmenata u kolu. Posmatrajmo prvi sluaj: akumulatori su vezani redno u s segmenata, a segmenti su
povezani paralelno. Tada je jaina struje
nE E
I1 = =
rn R rn
R+ +
s n N
Jaina struje u ovom sluaju (I1) ima maksimalnu vrijednost ako je imenilac u gornjem razlomku minimalan.
r R
Prema tome, problem se svodi na odreivanje minimalne vrijednosti funkcije y (n) = n +
N n
Na osnovu jednaine (3) imamo da je
r
yMIN = 2 R
N
Prema tome
E E N
I1MAX = = = 20 A.
rR 2 rR
2
N
U drugom sluaju, kada su akumulatori paralelno vezani u s redno vezanih segmenata, imamo da je
N
E
sE NE
I2 = = n =
r rN rN
R+s R+ 2 Rn +
n n n
Slinim postupkom dobijamo da je I1MAX = I 2MAX = 20 A
Prema tome, u ovom sluaju nije mogue dobiti jainu struje veu od 20 A.
Umjesto zakljuka
Ovdje je naveden jedan od nekoliko naina odreivanja maksimuma i minimuma bez korienja infinitezimalnog rauna.
Ovaj nain je ilustrovan odgovarajuim primjerima iz srednjokolske fizike.
Primjena razmatranog naina/metode omoguava rjeavanje navedenih i njima slinih zadataka bez korienja
matematikog aparata tzv. vie matematike. Zbog toga je korisno da ga uenik upozna, a naroito oni koji se pripremaju
za dravno ili meunarodno takmienje iz fizike.
Radovan Ognjanovi

23
Paradoksi i sofizmi

Paradoksi i sofizmi
Ova rubrika e vas u svakom broju upoznavati sa paradoksima koji su dovodili do manjih ili veih
kriza u matematici, kao i sa sofizmima koji imaju za cilj da naue uenike da izbjegavaju lane
zakljuke i nepravilnu upotrebu rijei.

Ureuje Mihailo Radevi II10

PARADOKSI
Najire shvaeno, paradoks (ili antinomija) je rasuivanje Matematika je uspravno zdanje koje bi se,
koje vodi u protivrjenost, iako izgleda da su polazne za razliku od neke visoke zgrade, sruilo ako
postavke tane, a pravila rasuivanja ispravna.
bi makar samo jedna matematika cigla bila
Kroz istoriju, paradoksi su znali da dovedu do razvoja rava. Dopustite li da se u sistemu pojavi makar
odreenih matematikih disciplina. Tako je, na primjer, i najsitnija greka, vie se ni u ta ne moete
matematiar Bertrand Rasel1, poetkom XX vijeka, u pouzdati. Prema jednoj logikoj teoremi, ako u
naivnoj teoriji skupova otkrio paradoks koji je doveo neki logiki sistem ue neka pogrena teorema,
do naglog razvoja matematike logike, to je svakako
onda biste pomou nje mogli dokazati da je 1
uticalo kako na modernu matematiku, tako i na logiku kao
filosofsku disciplinu. jednako 2.

Raselov paradoks

U Kantorovoj naivnoj teoriji skupova, pod skupom se


podrazumijeva kolekcija objekata koji imaju zajednik
svojstvo. Dugo se smatralo da je svaki skup definisan na
ovaj nain ispravno definisan skup. Tada je Rasel postavio
pitanje koje je ponitilo ovo uvjerenje. Pitanje je glasilo: Anegdota:
Da li skup svih skupova koji ne sadre sebe, sadri sebe?.
Prilikom jednog razgovora, neki skeptik pokuao je
Skup kao svoje elemente moe da sadri druge skupove. da nadmudri logiara Bertranda Rasela tako to bi
Na primjer, skup A = {{1, 2, 3} ,{1, 2} ,{5, 6, 7}} sadri tri osporio ve pomenutu logiku teoremu.
skupa. Dalje, skup moe da sadri i sam sebe. Evo jedne U redu, kazao je skeptik, ako dopustim da je jedan
takve definicije: Skup svih skupova koji imaju vie od jednako dva, dokaite mi onda da ste vi papa.
dva elementa. Poto takvih skupova ima vie od dva, i on
sam e sadrati vie od dva elementa, pa e biti dio samog Pria se da je Rasel samo asak razmiljao, a onda
sebe uzvratio ovako:
Definiimo skup R2 kao skup svih skupova koji ne sadre Papa i ja smo dva, prema tome papa i ja smo
sebe. Postavimo pitanje, da li R sadri sam sebe? Pretpo- jedan.
stavimo da R ne sadri sebe. Poto je R skup svih skupova
Iz knjige Euklidov prozor Leonarda Mlodinova
koji ne sadre sebe, trebalo bi ipak da sadri sam sebe,
to je kontradikcija sa poetnom pretpostavkom. To znai
da moramo pretpostaviti suprotno da R ipak sadri sebe.
Meutim, ni to nije mogue jer tada, po definiciji, R ne bi
sadrao sebe.
1
Bertrand Arthur Wiliam Russel (1872 - 1970)
2
obiaj je u savremenoj matematici da se u ast Rasela taj skup
oznaava poetnim slovom njegovog imena

24
Paradoksi i sofizmi
Radi boljeg shvatanja Raselovog paradoksa, formuliimo sljedee paradokse koji su po strukturi isti kao i Raselov:

Paradoks brice
U puku je bio vojnik kojemu su naredili da brije sve one i samo one vojnike iz svog voda koji se ne briju sami. Postavlja
se pitanje kako e se on odnositi prema samom sebi. Ako se bude brijao, treba ga uvrstiti u one koji se briju sami, a te
vojnike on ne smije brijati. Ako se on sam ne bude brijao, treba ga prikljuiti vojnicima koji se sami ne briju, a tada je
po naredbi duan da se obrije.

Paradoks laljivca - Epimenidesov paradoks (potie jo iz antikog vremena)


Posmatrajmo reenicu Ja laem. Ako je ona istinita, onda ja laem, pa je ta reenica lana. Ako je ona lana, onda ja
ne laem, pa je ona istinita.

U principu, paradoksi se mogu javiti iz dva razloga: kao posljedice neodgovarajuih pretpostavki odreene matematike
teorije i usljed nedovoljne razvijenosti nae intuicije. Raselov paradoks odnosi se na prvi sluaj i prevazien je uvoenjem
nove aksiomatizacije teorije skupova1

SOFIZMI
Sofizam je namjerno nemogu dokaz, laan dokaz nekog tvrenja. Pri tome se greka vrlo vjeto i iskusno maskira u
nekom dijelu dokazivanja.

Rijeiti sofizam znai pronai greku u rasuivanju na osnovu koje je zasnovan sofizam. Sofizmi imaju za cilj da
pomau kritikom miljenju jer ne treba se samo drati odreene logike sheme, odreenih misaonih procesa, ve i
kritiki osmisliti svaki korak rasuivanja usvojenim principima matematikog miljenja i raunske prakse.

Primjer:

Posmatrajmo niz jednakosti:


A = B;
A2 = B 2 ;
A2 = A B (jer je A = B );
A2 B 2 = AB B 2 ;
( A B ) ( A + B ) = B ( A B );
A + B = B;
B + B = B (jer je A = B );
2 B = B;
2 = 1.
Gdje je greka?
Odgovor je na strani sa reenjima.

Literatura:
1. <http://www.im.ns.ac.yu/personal/madaraszr/Rozi-licno/skupovi.pdf>
2. Mio Mileti, Paradoksi i sofizmi u matematici, Arhimedes, Beograd, 2004.
3. Milan Boi, Pregled istorije i filozofije matematike, Zavod za udbenike i nastavna sredstva, Beograd, 2002.
4. N.J.Vilenkin, Prie o skupovima, kolska knjiga, Zagreb, 1975.

1
uveden je ZF sistem aksioma teorije skupova (nazvana po poetnim slovima prezimena matematiara Zermela (Ernst Friedrich
Ferdinand Zermelo 1871-1953) i Frenkela (Adolf Abraham Halevi Fraenkel, 1891-1965) koji su ga uveli.

25
Zanimljiva matematika - Brojevi

Mirakuli
Iz referata Magije brojeva Lilande orovi i Jelene Dakovi
I10 (kolska 2006/07)
Pri raunskim operacijama sa brojevima mogu se dobiti rezultati koji izazivaju i uenje i divljenje, poznati kao
mirakuli1. Mirakuli koji slijede odnose se na raun sa brojem 9. Pogledajte:

1 9 + 2 = 11 12345679 (1 9) = 111111111 9 9 + 7 = 88
12 9 + 3 = 111 12345679 (2 9) = 222222222 98 9 + 6 = 888
123 9 + 4 = 1111 12345679 (3 9) = 333333333 987 9 + 5 = 8888
1234 9 + 5 = 11111 12345679 (4 9) = 444444444 9876 9 + 4 = 88888
12345 9 + 6 = 111111 12345679 (5 9) = 555555555 98765 9 + 3 = 888888
123456 9 + 7 = 1111111 12345679 (6 9) = 666666666 987654 9 + 2 = 8888888
1234567 9 + 8 = 11111111 12345679 (7 9) = 777777777 9876543 9 + 1 = 88888888
12345678 9 + 9 = 111111111 12345679 (8 9) = 888888888 98765432 9 + 0 = 888888888
123456789 9 + 10 = 1111111111 12345679 (9 9) = 999999999

Teko je rei koji je mirakul impresivniji. Vjerovatno se pitate u emu je tajna. Kako u matematici nita nije sluajno,
tajna je u sljedeem:

Prvi mirakul
Posmatrajmo izraz:
(10 n 1
+ 2 10n 2 + 3 10n 3 + ... + n ) (10 1) + ( n + 1) , n {1, 2,..., 9}.
Lako se provjerava da za n {1, 2,..., 9} dobijamo redom prvi, drugi, ..., deveti red na lijevoj strani. Kada se izvri
mnoenje i slini sabirci saberu, dobija se
10n + 10n 1 + 10n 2 + ... + 10 + 1 = 111
...
1
n +1

to predstavlja izraz na desnoj strani.

Drugi mirakul
Prva jednakost se direktno provjerava, a ostale slijede iz nje, s obzirom na to da je
12345679 (9 k ) = k 111111111 = kkkkkkkkk za k = 2, 3,..., 9.

Trei mirakul
Posmatrajmo izraz:
(9 10 n 1
+ 8 10n 2 + 7 10n 3 + ... + (10 n) ) (10 1) + (8 n), n {1, 2,...,8}
Slino kao u prethodnom primjeru prvi, drugi,..., osmi red na lijevoj strani dobijaju se redom za n = 1, 2, 3, ..., 8. Nakon
izvrenih operacija mnoenja i sabiranja, dobijamo

( )
9 10n 10n 1 10n 2 ... 2 = 9 10n 10n 1 + 10n 2 + ... + 2 = 9 10n 11
= ...
1... ,
n11 12 888
n +1 8

to predstavlja izraz na desnoj strani.

1
od engleske rijei miracle to znai udo

26
Zanimljiva matematika

ibice
1. Od 29 tapia, iste duine, sastavljena je netana jednakost. Premjestite samo jedan tapi tako da se dobije
tana jednakost.

2. Ova jednaina rimskih brojeva sasvim je tana, pa ne morate da pomijerate ni jednu ibicu. Kako to?

3. Petnaest ibica rasporeeno je tako da formira pet kvadrata. Oduzimanjem tri ibice treba dobiti tri kvadrata.

4. Datih devet ibica ine tri jednakostranina trougla. Premjestiti samo dvije ibice tako da se dobiju dva
jenakostranina trougla.

Pripremile
Ivana Popovi, IIIl
Jovana Popovi, I12

27
Zanimljiva matematika
IZBOR REDAKCIJE

Zanimljiva matematika
Ljepljivi smajliji Tajanstveni niz
Milan, Boidar i Ivana bacaju ljepljive smajlije na tablu. Koja cifra nedostaje u nizu: 6 2 5 5 4 5 6 3?
Svaki smajli koji bace osvaja 10, 13 ili 17 poena. Tabela (Autor ovog zadatka je izjavio: Ne poznajem nikoga ko
pokazuje koliko je kojih smajlija baeno na tablu, ali ne je mogao da rijei ovu zagonetku!) Moete li vi?
znamo koliko je kojih bacila svaka osoba. Znate ovo: Ivana
ima 56 poena. Broj Boidarovih poena je dvocifren broj
sa dvije iste cifre. Boidar ima tri poena vie od Milana.
Naite uljeza
Koliko je kojih smajlija svaka osoba bacila na tablu? Koji se od ovih brojeva ovdje ne uklapa?
3 8 15 24 35 48 54 63
10 poena
13 poena ta predstavlja slika?
17 poena

Krivudavi zatvor
Kako ete lako ustanoviti da li se mrav nalazi unutar ili
izvan krivudavog zatvora a da ne pratite trag do izlaza ?

U pitanju je fraktalni1 iks-oks Patrika Grima i Pola Sen


Denija
Na slici, matematiari Grim i Sen Deni nude analitiku
prezentaciju svih moguih partija igre iks-oks. Svako polje
iks-oksa podijeljeno je na manje table da bi se prikazali
Elegantne elipse razliiti mogui izbori.
Nacrtati etiri prave linije koje e podijeliti ravan na takav
nain da svaka elipsa bude smjetena u odvojenu oblast.
Iz jednog poteza
U jednom dokazu Pitagorine teoreme koristi se sledea
slika
Da li moete da je nacrtate
jednim potezom olovke, ne
podiui njen vrh od papira
ne idui nijednom od dui dva
puta?

Tri kvadrata Datum


U kvadratima se nalaze brojevi od 1 do 12. Zbir brojeva U Nekom mjesecu tri utorka su imala paran datum. Koji je
prvog kvadrata je 24, drugog 34 a treeg 20. Dopunite datum bio poslednjeg petka u tom mjesecu?
brojeve koji nedostaju.
1 ? 8 ? 6 ? 1
Fraktali - slike nastale iteracijom, tj. upornim uzastopnim
? 11 ? 5 ? 3 ponavljanjem nekog raunskog ili geometrijskog
postupka.

28
Zanimljiva fizika

Pitanja i odgovori 10+10


Priprema Radovan Ognjanovi

dvaput veom brzinom od brzine kretanja ose cilindra.


Od ovog broja objavljujemo pitanja i odgo- Zbog toga, kada ovjek pree 10 metara, onda e osa
cilindra biti ispred njega na rastojanju od 5 metara.
vore koji sadre razliite kvalitativne zadatke
Kada pree jo 5 metara, osa cilindra e mu biti na
i probleme, paradokse, manje ili vie rastojanju od 2,5 metara itd. Hoe li ovjek uopte i
poznate podatke, injenice, pojmove i zakone doi do cilindra?
fizike. Akcenat e biti na kvalitativnoj strani
posmatrane pojave. Matematiari rjeenje ovog zadatka obino svode na
izraunavanje granine vrijednosti zbira beskonano
Fizika i pitanja kojima se bave fiziari nijesu mnogo razlomaka iji je brojilac duina daske, a
ogranieni zidovima kabineta i laboratorija, imenilac redom brojevi: 1, 2, 4, 8, 16, 32, ...
ve su neprestano prisutni u svijetu u kojem 3. Talasna duina crvene svjetlosti u vodi jednaka je
postojimo. talasnoj duini zelene svjetlosti u vazduhu. Kakvu
Ovdje se ne tei saoptavanju novih znanja ve boju vidi ovjek pod vodom kada je voda osvijetljena
crvenom svjetlou?
osvjeavanju i oivljavanju osnovnih znanja iz
fizike, koje italac ve ima. Za pitanja i odgovore 4. Lopta je izbaena vertikalno navie. Koje je vrijeme
iz neobine fizike obinih pojava nije bitno due: penjanja ili sputanja?
koliko e pitanja italac uspjeno odgovoriti, ve 5. Kondukter u vozu hodao je od prvog do zadnjeg
koliko e vremena razmiljati o njima. Naravno, vagona i vratio se nazad, do prvog vagona. Intenzitet
umjena primjena ovih pitanja i odgovora u brzine voza je u toku ukupnog kretanja konduktera
nastavi fizike pobuuju interesovanje za fiziku i vei ili jednak intenzitetu brzine konduktera. Da li
podrava aktivnije usvajanje znanja. je kondukter preao dui put u odnosu na zemlju od
putnika koji mirno sjedi?
U svakom broju e biti po 10 novih pitanja
i odgovora. Nadamo se da e vam pitanja iz 6. Dva jednaka izdubljena sferna ogledala postavljena su
zanimljive fizike ujedno biti zanimljiva pitanja jedno pored drugog tako da im se glavne optike ose
i ie poklapaju. Takasti izvor svjetlosti postavljen je
iz fizike.
na zajednikoj optikoj osi na rastojanju d od jednog
ogledala. Gdje se dobija lik poslije odbijanja zraka od
1. Mogu li se privlaiti dva istoimeno naelektrisana oba ogledala?
tijela?
7. Kolikom silom treba vui ue pri dizanju tereta teine
2. Jedan kraj daske duine 10 metara je na cilindru, a Q = 100N pomou nepokretnog kotura, a kolikom ako
drugi kraj dri ovjek, kao na slici. On je gura naprijed se koristi pokretni kotur? Pretpostavlja se da su teine
a cilindar se kotrlja bez proklizavanja po horizontalnoj koturova dovoljno male da se mogu zanemariti.
podlozi. Osim toga, nema ni klizanja izmeu daske i
cilindra. Daska je sve vrijeme horizontalna. Koliko 8. Kada nam je Sunce blie: ljeti ili zimi?
rastojanje treba da pree ovjek dok drugi kraj daske 9. Kako moete u toku jedne sekunde da smanjite svoju
ne doe do cilindra? teinu?
10. Jedan kraj ice je uvren za plafon, a za drugi kraj
je privezano tijelo. Pustimo tijelo da slobodno pada.
Pod dejstvom tijela ica e se izduiti. Poznato je da
se potencijalna energija tijela smanji za E, a u isto
Napomena: vrijeme, potencijalna energija ice se povea samo za
Poto se cilindar kotrlja bez klizanja po podlozi, E / 2. Da li je ovo u saglasnosti sa zakonom odranja
onda se du kojom cilindar dodiruje dasku kree se energije?

29
Logike igre

Sudoku
Sigurno ste, ako ne probali da rjeavate, onda bar uli za se sastoji od devet kvadratia. U svaki kvadrati treba
sudoku. Rije je o igri koja brzo i uspjeno osvaja svijet upisati po jednu cifru od 1 do 9, tako da svaki red, kolona
zahvaljujui svojoj zaraznoj drai logikih zagonetki i svaki boks sadre devet razliitih cifara. Neki kvadratii
koje vas mogu drati okupiranim satima. Smatra se da ju ve sadre brojeve, unaprijed zadate, koji jednoznano
je izmislio uveni vajcarski matematiar Leonard Ojler odreuju poloaj svih ostalih brojeva u reetki.
(1707-1783). Meutim, tokom osamdesetih ova igra
postala je svojevrsna opsesija Japanaca koji je nazivaju Postoje dvije osnovne metode koje pomau pri izradi
sudoku (su = broj, doku = po jedan). Sudoku je veoma sudokua: slajsing-end-dajsing i brojanje. Obje metode
popularan zbog relativno jednostavnih pravila. Tok misli zasnivaju se na metodi eliminacije.
da bi se zagonetka rijeila do kraja moe postati veoma
komplikovan, ali u tome je ar sudokua. Teina zagonetke Slajsing-End-Dajsing
zavisi od toga koliko je lako logiki utvrditi brojeve koji
nedostaju Ovom metodom eliminiu se sve kolone i redovi u jednom
boksu u kojima se ve nalazi broj ija se pozicija utvruje
Prvo emo prei preko osnovnih metoda rjeavanja (broj 3 na primjeru sa slike desno).
sudokua a za one koji nakon toga budu zainteresovani za
ovu igru spremili smo par problema. Strpljenje i vjeba
su dva kljuna momenta u dugoj i uspjenoj karijeri 3
rjeavanja sudokua.

Metode rjeavanja
Za rjeavanje tipinog sudokua nije potrebno apsolutno
nikakvo znanje iz matematike, osim sposobnosti da brojite
od 1 do 9. Izazov je u tome da se brojevi stave na njihovo 3
tano mjesto u reetki.

1 2 3 4 5 6 7 8 9 3
2 5
3
3
4 1 6 5
5 9 3 7
Prvo pregledate sve redove (slajsing), a onda provjerite
6 2 8 4 kolone (dajsing). Vano je provjeriti sve cifre, i pregledati
brojeve po redoslijedu njihove uestalosti.
7
U datom primjeru analizira se pozicija broja 3 u donjem
8 boks lijevom boksu. Ako pogledamo redove vidjeemo da
se broj 3 ne moe nalaziti u poslednjem, devetom redu.
9 Takoe ne moe biti ni u prvoj ni drugoj koloni to
ostavlja samo dva mogua polja u kojima bi se broj 3
Standardna sudoku reetka sadri 81 kvadrati i podijeljena mogao nalaziti. Poto se u jednom od ta dva polja nalazi
je u devet horizontalnih redova, devet vertikalnih kolona broj 5, pozicija broja 3 jednoznano je odreena u donjem
i devet bokseva ili regiona. Svaki od devet bokseva lijevom boksu - moete ga sa sigurnou upisati.

30
Logike igre
Brojanje U datom primjeru broj 1 se u lijevom srednjem boksu
moe nalaziti na jednom od dva mjesta (do ovog zakljuka
Druga tehnika, brojanje, podrazumjeva pregled boksa, lako se dolazi metodom slajsing-end-dajsing) to znai da
redova i kolona da bi se otkrili brojevi koji nedostaju. se nigdje drugo, u istoj koloni, ne moe nalaziti broj 1.
Vrijednost pojedinanog kvadratia ponekad se moe To nam omoguava da sa sigurnou upiemo broj 1 u
otkriti i brojanjem u nazad, tj. pregledom njenog boksa, osijeneni kvadrati prvog boksa.
reda i kolone za brojeve koji bi potencijalno mogli da
odgovaraju i onda vidjeti koji je broj preostao. Druge, napredne metode eliminacije brojeva kandidata da
bi se dolo do pravog rjeenja objasniemo u sljedeem
broju, a vama ostavljamo da se, sada kada ste opremljeni
1 osnovnim znanjem, zabavite rjeavanjem zagonetki koje
smo vam pripremili:
2
Laka
6
4 5 7 8 9 1 2 7 6
3 1 3 5
5 6 4 8
8 3 2 1
9 4 5 8 6
U ovom primjeru u osjenenom kvadratiu moe biti 7 2 4
samo broj 4, jer su sve druge mogunosti iskoriene - u 9 2 4 5
njegovom redu nalaze se brojevi 5 i 7, u koloni brojevi 1,
2, 8 i 9 a u njegovom boksu 3 i 6. 6 8 9
4 8 9 7 1 3
Brojevi kandidati
Tea
Kada zagonetka postane tea, bie potrebno zabiljeiti
brojeve kandidate (brojeve na koje ste suzili pretragu
za odreeni prazni kvadrati, u primjeru sa desne strane 9 7 4
nalaze se u svjetlijim kvadratiima). Poslije unoenja 6 5
definitivnih brojeva, lako moete otkriti i zapisati brojeve
7 6 1
kandidate za svaki kvadrati.
4 9 2
| | 1 9 7 4 8 6
6 5 1 3 7 1
| | 7 6 1 3 5 1
4 1
9 2 9 5
| 1
8 6 1 8 7
| 3 7 1
Savjet:
| 3 5 1
| 9 5 - Koristite obinu olovku,
1 8 7 - Nikada ne pogaajte! Uvijek pravite poteze bazirane na
logikoj dedukciji.
Ako uvidite da ste sveli jedan broj u isti red (ili kolonu) i
boks, moete primijeniti metod eliminacije u fazi slajsing- Bojana estovi,
end-dajsing i brojanje da biste jo vie suzili pretragu. Gordana Boljevi,
IV-h
31
Zanimljiva matematika
IZBOR REDAKCIJE

Matematiko-logike zagonetke i problemi

1 Gdje je euro?
Za ruak u kafani tri prijatelja 2 Kako dobiti auto?
platila su kelneru po 5 eura.
Blagajnik za kasom rekao je Tri brata treba da podijele 35 automobila
kelneru da ruak ukupno vrijedi iste klase koje im je otac ostavio u nasljee,
10 eura i dao je kelneru 5 eura i to tako da najstariji brat dobije 1/2, srednji
nazad. Kelner je svakom od 1/3, a najmlai 1/9 svih automobila. Budui
trojice gostiju vratio po jedan i da takvu podjelu nijesu mogli da izvre jer
za sebe zadrao 2 eura. Dakle, 35 nije djeljiv sa 2, 3 i 9, braa su potraila
svakog gosta ruak je kotao 4 pomo od svog prijatelja matematiara. On je
eura, to iznosi 12 eura, i ako se u podjelu ukljuio i svoj automobil, tako da
jo dodaju 2 eura koja je kelner je sada na raspolaganju imao 36 automobila
zadrao za sebe, to ukupno iznosi koje je podijelio prema testamentu. Prvi je
14 eura Gdje se izgubio jedan dobio 18, drugi 12, a trei 4 automobila.
euro? To je ukupno 34, pa je matematiar uzeo
svoj automobil, a braa, zadovoljna to
su podjelom dobili vie nego to im po
testamentu pripada, poklonie mu i drugi
3 Ko lae? automobil. Da li je, ipak, sve u redu ?

Dvoje djece, djeak i djevojica,


meusobno razgovaraju:
Ja sam djeak, kae dijete
sa crnom kosom 4
Sijalica i tri prekidaa
Ja sam djevojica, kae dijete
Nalazite se ispred vrata sobe u
sa riom kosom.
kojoj se nalazi samo jedna sijalica
Ako se zna da bar jedno od njih Isred Vas su tri prekidaa. Samo
lae, ko je ko? jedan prekida pali svijetlo. Vi ne
vidite ta se deava u sobi, poto
su vrata zatvorena. Imate pravo
samo jednom da uete u sobu,
pritom u sobi moete da radite
ta hoete i ne moete izlaziti
5 Tri gimnastiara napolje i vraati se opet. Kako
ete odrediti koji prekida pali
Tri gimnastiara imaju na sijalicu?
majicama takmiarke brojeve 1,
3 i 6. Kakav poloaj oni treba
da zauzmu da dobijeni trocifren
broj bude djeljiv sa 7 ?

32
Zanimljiva matematika

Ajntajnov zadatak
Zadatak je postavio Albert Ajntajn, tvrdei da 98% ljudi nije u stanju da ga rijei. Da li ste meu onih 2%?

U jednom naselju nalazi se pet susjednih kua u pet razliitih boja. Vlasnik svake od njih druge je nacionalnosti. Svaki
od tih ljudi pije drugaije pie, pui cigarete razliite marke i ima drugaijeg kunog ljubimca.

Koji od njih dri zlatnu ribicu?

Uputstva:
- Englez ivi u crvenoj kui.
- veanin ima psa.
- Danac pije aj.
- Zelena kua je desno od bijele i jedna su do druge.
- Vlasnik zelene kue pije kafu.
- ovjek koji pui cigarete Pall-Mall dri ptice.
- Vlasnik ute kue pui cigarete Dunhil.
- ovjek koji stanuje u kui u sredini pije mlijeko.
- Norveanin ivi u prvoj kui.
- ovjek koji pui cigarete marke Blend, stanuje pored ovjekakoji ima make.
- ovjek koji dri konje stanuje pored ovjeka koji pui cigarete Dunhil.
- Onaj koji pui cigarete Blue Master pije pivo.
- Njemac pui cigarete Prince.
- Norveanin stanuje do plave kue.
- Susjed ovjeka koji pui cigarete Blend pije samo vodu...
...rjeenje u sljedeem broju.

33
Citati

Matematika je klju za cjelokupno ljudsko znanje

Vi koji uite, uite matematiku.


Ne gradite bez temelja"

Matematiku Leonard Ojler


treba uiti zato to ona um u
harmoniju dovodi"
Leonardo Da Vini

Nikada neemo
postati matematiari, ak i ako
znamo napamet sve tue dokaze, ako na um
nije sposoban da samostalno rjeava bilo
kakve probleme"

Priroda je
Mihail Vasiljevi Lomonosov
ogromna knjiga u kojoj je
napisana nauka. Ona je stalno otvorena
pred naim oima,ali ovjek je ne moe razumeti
ukoliko prethodno ne naui jezik i slova kojima
je napisana. A napisana je ona jezikom
Rene Dekart matematike..."

Postoji jo jedan
razlog zbog koga matematiku treba
posebno cijeniti. Upravo ona daje prirodnim naukama
stepen pouzdanosti koji se bez nje
ne bi mogao dostii"
Galileo Galilej

Albert Ajntajn
34
Zanimljiva fizika

Nikola Tesla i njegov prvi susret sa


Z a n imljiva elektricitetom

f i zika
Mnogi kau da se sa nekim stvarima iz djetinjstva
susrijeemo cio ivot. Izgleda da se to desilo i Nikoli Tesli.
Kako je izgledao njegov prvi susret sa elektricitetom,
najbolje je opisao on sam:
Bio je suton. Pomilovao sam svog maka i tada se
urka poznatih fiziara dogodilo udo od koga sam zanijemio. Moja ruka je
izazvala pljusak praskavih varnica, a iznad makovih lea
Poznati svjetski fiziari odluili su da pou na zajedniki se stvorilo polje svjetlosti.
ruak. Posmatrajui goste, vratar, koji je bio student fizike,
zapazio je sljedee: Teslin otac Milutin Tesla, smatrao je da je to ona ista
pojava koja se stvara pri udaru groma, elektricitet.
Ajntajn je smatrao da je to bilo relativno dobro vrijeme za Na kraju svega, svakako je najzanimljiviji zakljuak
ruak. povodom ovog dogaaja. Nikola Tesla je tada postavio
Svi su gravitirali prema Njutnu, ali se on neprekidno kretao u pitanje na koje e samo on znati da odgovori:
krug konstantnom brzinom i nije pokazivao reakciju A ja sam oaran razmiljao: - Nije li priroda jedna ogromna
Paskal je bio pod suvie velikim pritiskom da bi mogao da maka? Ako jeste, ko nju miluje po leima? Tada sam imao
uiva. tri godine a uda koja su dolazila bila su sve vea.
udesni maak je bio prijatelj malog Nikole, bili su
Om je proveo najvei dio vremena pruajui otpor nerazdvojni. Moda je i na taj nain priroda htjela da
Amperovom miljenju o toku dogaaja. pokae kako e otkrivati svoje najvee tajne onima koji
Volt je mislio da prisutno drutvo ima veliki potencijal budu voljeli i cijenili podjednako i ljude i ivotinje kao to
je to inio Nikola Tesla.
Hajzenberg je mogao i da bude i da ne bude tamo Iz knjige Tesla, mladost Vladimira Pitala
De Brojli je stajao u uglu i talasao se
Sve je magnetski privlaila Teslina linost. Plava tenost
Ispostavilo se da je Vat snaan govornik. Ludi naunik zuri u boicu plave tenosti. Ova supstanca
Herc je odlazio do stola u bifeu nekoliko puta u minuti ostaje tena na 40, rekao je asistentu Borisu.
Boris je upitao: Celzijusovih ili Farenhajtovih?
Faradej je bio veliki kapacitet za hranu. Svojim tamnim oima, naunik je pogledao Borisa i rekao:
Nije vano.
Pripremila:
Zato je naunik to rekao?
Irena Mara,
Sami zakljuite ili, ako ste nestrpljivi, pogledajte rjeenje
IV-j na strani RJEENJA
35
Intervju

U razgovoru sa Beom
Razgovarali smo sa uenikom nae kole Beom Meruliem dan prie polaska u Makedoniu na Balkansku Matematiku
Olimpiadu. Ova mladi matematiar ve e uspeno uestvovao na raznim takmienjima:

: Zato ba matematika?
Beo: Za matematiku sam se odluio u estom razredu osnovne kole, er e to bio edini predmet iz koeg e bilo mogue
takmiiti se tada. Na Regionalnom takmienju bio sam prvi i plasirao se za Republiko, na koem sam osvoio drugo
mesto, pa e usliedilo Savezno takmienje. Uspeh godi. Rieio sam da nastavim da se takmiim iz matematike.
: Da li e smisao za matematiku rezultat rada ili talenat?
Beo: Talenat e, naravno, potreban. I nema ga svako. Smatram da ona ko
nema talenta ne moe da postigne uspeh na takmienju. Naravno, potrebno
e i mnogo rada.
: Kako na uenika u vezi s tim utie nastavnik?
Beo: U osnovno koli mi e predavao saan nastavnik, koi e matematiku
uviek inio interesantanom i birao zanimljive zadatke. To e, svakako,
doprinielo da zavolim ova predmet.
: U osmom razredu si nakon Republikog takmienja otiao na
Balkaniadu, takmienje na koem su uestvovala 72 uenika iz 12 zemalja.
Koliko ti znai bronzana medalja kou si osvoio tada u Bugarsko?
Beo: Naravno da mi znai mnogo. Bilo e to nezaboravno iskustvo. Medalja
e podstrek za dalje dokazivanje, za vie rada. Onda ni uspeh ne izostae.
: Tvoe ovogodinje uee na takmienju potvrue da se nee zadovoljiti
postignutim. Da li tvoe ambicie prevazilaze balkansko takmienje?
Beo: Naravno da Balkaniada nie mo kranji cilj. To e bilo veoma vano takmienje, ali nadam se da nee biti i
navee.
: Koe su glavne teme razgovora kada se na ednom mestu nae toliko talentovanih, mladih matematiara?
Beo: Razgovarali smo o takmienjima, zadacima, nainu vebanja. Sporazumievanje nie bilo problem. Stekao sam
nove priatelje.
: ta te e motivisalo da krene na kolsko takmienje u Gimnazii?
Beo: Motivie me elja da nastavim uspeh iz osnovne kole.
: Da li e osvoeno prvo mesto znailo vie obaveza i nove, naporne pripreme za Republiko takmienje?
Beo: Za Republiko takmienje niesam se dugo spremao. Bilo e u vrieme ozbiljnih obaveza iz drugih predmeta.
Sedmicu pred takmienje bio sam slobodan, to sam iskoristio da vebam. Pomogla mi e, naravno, profesorica, koa
mi e dala nekoliko zbirki zadataka.
: Prole godine e NVO Detinjstvo traila na diete iz razliitih oblasti. Ti si odabran za na diete nauke.
Verovatno si ponosan.
Beo: Naravno da sam ponosan. Uspeh svakom pria i rad ini lakim.
Kada smo Bea pitali ta e studirati, odgovorio nam e da o uviek razmilja, ali e to verovatno biti - ekonomia. Za
kra smo ga zamolili da matematiku opie ednom rieu. Najprije tiina. Beo konstatue da e to veoma teko. Ali
uspeno nalazi reenje zadatka i odgovara: razmiljanje. Sloili smo se. Razmislite i vi o tome.
Od Bea smo se rastali uz elju da na Balkaniadi osvoi zlato. Znamo da e to, ako se ne desi ove, sigurno biti sljedee
godine. Bie to povod da ponovo razgovaramo s njim o novim uspesima.
Milan Lakievi,
Boidar Bukili
III - L
36
Odabrani zadaci

Odabrani zadaci
S lakoom uraditi ono to je drugima teko, eto talenta;
uiniti ono to je nemogue talentu, eto genija!
Anri Frederik Amijel
Profesori sa PMF-a biraju za vas:
Za one koji vole da rjeavaju nestandardne zadatke iz matematike, rubrika Odabrani zadaci e
svakim brojem donositi nove zadatke iz raznih oblasti matematike.
Svi itaoci mogu se takmiiti u rjeavanju odabranih zadataka. Rjeenja zadataka slati otkucana ili
itko ispisana na adresu ili e-mail redakcije asopisa, najkasnije do 01. maja 2009. godine. U sljedeem
broju nagrauju se tri takmiara koja su bila najuspjenija u rjeavanju odabranih zadataka.

1. U prvi razred policijske srednje kole upisano je 2. Tablica 5 5 puni se brojevima iz skupa {-1, 0, 1},
n uenika. Na prvom asu gimnastike, uenici su a zatim se izraunaju sume po vrstama, kolonama i
postrojeni u jedan red i okrenuti licem ka nastavniku, na obje dijagonale. Dokazati da bez obzira na nain
kao na slici 1 popunjavanja tablice meu dobijenim sumama postoje
bar dvije jednake.
3. Postoji li prirodan broj n tako da je broj
slika 1. stanje prije komande NALIJE-VO k = 1 + n + n 2 + n3 + ... + n 2008
djeljiv sa 2008? Odgovor obrazloiti.
Na njegovu komandu NALIJEVO, dio uenika se
okrene nalijevo a dio uenika se, zbog treme, okrene 4. Odrediti sve prirodne brojeve koji su za 2008 vei od
nadesno (npr. kao na slici 2). zbira kvadrata svojih cifara.

5. Dokazati da vai 2 + 3 3 + ... + 2008 2008 < 2


6. Nai minimum funkcije:
slika 2 - jedno mogue stanje poslije komande
NALIJE-VO f ( x) = x 2 4 x + 8 + x 2 10 x + 41
Za koje se vrijednosti dostie taj minimum?
Shvativi greku, u sljedeoj sekundi svi uenici koji 7. Na pravoj su date:
gledaju jedan u drugog i susjedi su, okreu se za 180
stepeni (vidi sliku 3). a) tri take A, B i C; b) etiri take A, B, C i D.
Odrediti na pravoj taku T tako da zbir rastojanja od
take T do datih taaka bude minimalan.
8. Dat je pravougaonik ABCD. Dvije krunice konstru-
slika 3. stanje poslije prve sekunde
isane su tako da jedna od njih sadri take A i D
i dodiruje stranicu BC, a druga sadri take A i B i
Poslije jo jedne sekunde, opet se svi uenici koji
dodiruje stranicu CD. Dokazati da vai:
gledaju jedan u drugog i susjedi su okreu za 180
stepeni, pa imamo stanje kao na slici 4. 8(R1+R2) 5(AB+AD)
gdje su R1 i R2 poluprenici krunica.

slika 4. stanje poslije druge sekunde Adresa Redakcije


Gimnazija Slobodan kerovi,
Postupak se nastavlja u svakoj sekundi. Dokazati da e asopis za rubriku Odabrani zadaci,
se okreti u jednom trenutku zavriti, bez obzira koliki Vaka urovia 36, 81 000 Podgorica
je broj n i kako su se okrenuli uenici na komadu e-mail: gama.casopis@gmail.com
NALIJEVO.
Rjeenja zadataka bie objavljena u sljedeem broju.

37
Dravno takmienje

Dravno takmienje 2008


Matematika
Zadaci
I razred
0 1 13579
1. Neka je K = (135792 + 1) (135792 + 1)......(135792 + 1)
213580
13579 1
Dokazati da vai = 13578.
K
2. Poluprava iz tjemena A trougla ABC sijee stranicu BC u taki D, tako da je D izmeu taaka B i C, i sijee opisani
1 1 4
krug tog trougla u taki E. Dokazati da vai nejednakost: + .
AD DE BC
3. udesni 4 - Kalkulator ima etvorocifreni displej i etiri tastera. Kalkulator radi na sljedei nain:
- pritiskom na taster 1, broj na displeju se zamjenjuje sa 1.
- pritiskom na taster 2, broj na displeju se dijeli sa 2.
- pritiskom na taster 3, broj na displeju se smanji za 3.
- pritiskom na taster 4, broj na displeju se pomnoi sa 4.
Na poetku je na displeju 0. Svaka operacija koja za rezultat daje negativan broj, decimalni ili broj sa pet cifara, se
ignorie.
Da li se na displeju moe pojaviti broj 2008? Moe li se pojaviti 2010? Obrazloiti odgovore!

4. Neka je F preslikavanje takvo da za sve realne brojeve x i y vai F ( x y ) = F ( x) + F ( y )

1
= 2008, odrediti F ( 2008 ) .
2008
Ako je F
2008

II razred

1. Neka je skup nekih kompleksnih brojeva, koji ima vie od 5 elemenata. Ako za svako z S vai | z 2 | 2,
tada postoje dva razliita broja z i w iz S sa svojstvom | z w | 2 Dokazati

1 1 1
2. Ako su a, b i c tri razliita realna broja, onda su rjeenja jednaine + + = 0 dva razliita realna
xa xb xc
broja Dokazati

3. Neka su E i F bilo koje take konture nekog 2008-tougla. Dokazati da je duina dui EF manja ili jedanka duini
najvee dijagonale mnogougla.

4. U krunici radiusa r upisani su jednakostranini trougao i kvadrat sa jednim zajednikim tjemenom. Izraunati (u
funkciji od r) povrinu lika koji je zajedniki trouglu i kvadratu. Da li je ta povrina vea ili manja od polovine
povrine kruga?

38
Takmienja
III razred
1. Neka je a1, a2, ... , an, ... beskonaan niz realnih brojeva takav da za svaki prirodan broj n vai an+2 2an+1 +an =1

Ako su poznati lanovi a1 i a2 toga niza, izraziti lan an kao funkciju od a1, a2 i n (n je proizvoljan prirodan broj).

2. Dokazati da za proizvoljne pozitivne brojeve a, b, c i d vai najednakost

(a + b)(c + d ) + (a + c)(b + d ) + (a + d )(b + c) 6 4 abcd .

3. Neka je P(x) polinom sa cjelobrojnim koeficijentima. Na grafiku funkcije y = P(x) u koordinatnom sistemu xOy
uoene su dvije take A(x1, y1) i B(x2, y2), pri emu su njihove koordinate x1 i x2 cijeli brojevi. Ako je poznato da je
udaljenost izmeu taaka A i B cijeli broj, dokazati da je y1 = y2

4. U otrouglom trouglu ABC povuene su visine AA1, BB1 i CC1 (take A1, B1 i C1 redom pripadaju stranicama BC, CA
i AB). Ako su (ABC) i (A1B1C1) povrine trouglova ABC i A1B1C1, a , i uglovi trougla ABC, dokazati da vai

S ( A1B1C1 )
= 1 cos 2 cos 2 cos 2 .
S ( ABC )

IV razred
1. Osnova kvadra ABCDA1B1C1D1 je kvadrat. Odrediti najveu moguu veliinu ugla koji zaklapaju prava BD1 i ravan
BDC1

( )( ) ( )
3
2. Rijeiti jednainu x 3 2 2sin x 1 + 2 x 4 sin x = 0 u skupu realnih brojeva.

3. Neka su a, b, c i d realni brojevi takvi da je a b i c d. Cjelobrojna pravougaona reetka je skup


{( x, y ) | a x b, c y d , x Z , y Z } , gdje Z oznaava skup cijelih brojeva. Data je cjelobrojna pravougaona
reetka dimenzija 32008 (vidi sliku). Koliko postoji pravouglih trouglova ija su tjemena vorovi reetke?

.....
.....
.....
2008 kolona

4. Neka je N skup prorodnih brojeva i g: NN funkcija takva da je g(k) najvei neparni djelilac prirodnog broja k
n
g (k ) 2
Dokazati da za svaki prirodan broj n vai > n.
k =1 k 3

39
Takmienja

Rjeenja
I razred
1. Vai:

( 0

)( 1

)(
13578 K = 13578 135792 + 1 135792 + 1 135792 + 1 ... 135792
2

) ( 13579

)
+1 =

( 20
)( 20
)( 21
= 13579 1 13579 + 1 13579 + 1 13579 + 1 ... 13579 )( ) (
22 213579
+1 =)
= (13579 21
1) (13579 21
+ 1) (13579 22
+ 1) ... (13579 + 1) =
213579

(
= 135792
13579

)(
1 135792
13579

)
+ 1 = 135792
13580
1.

2. Iskoristimo nejednakost izmeu aritmetike i geometrijske sredine:


A
1 1 1 1 2
+ 2 = (1)
AD DE AD DE AD DE
Vai sljedee:

ADC BDE, kao unakrsni uglovi;


BED DCA, kao periferijski uglovi nad istim lukom BA
Odatle, na osnovu drugog stava slinosti trouglova slijedi
B D C
ADC ~ BDE

E
AD DC
=
BD DE

AD DE = BD DC.

Sada iz (1) dobijamo


1 1 2 2
+ = (2)
AD DE AD DE BD DC
Iskoristimo nejednakost izmeu aritmetike i geometrijske sredine

BD + DC BC
BD DC =
2 2

1 2
(3)
BD DC BC
1 1 4
Pa iz (2) i (3) dobijamo +
AD DE BC
3. 2008 se moe pojaviti kao rezultat na displeju. Na startu je 0. Na primjer pritiskom na 1 dobijamo 1. Zatim pritiskom
est puta na 4 dobijamo 4096, i najzad 696 puta biramo 3 i dobijamo 2008. 2010 se ne moe pojaviti kao rezulatat
rada Kalkulatora. Treba primijetiti da ako broj na displeju nije djeljiv sa 3, onda se nijednom operacijom ne moe
dobiti broj djeljiv sa 3. Naime, na poetku je 0 na displeju. Jedina operacija koja sada daje rezultat je biranje tastera
1. 1 nije djeljiv sa 3, pa nakon toga nijedan broj dobijen kao rezultat rada Kalkulatora nee biti djeljiv sa 3, a 2010
je djeljiv sa 3.

40
Takmienja
4.
1 1 1
F
2008
= F
2008 2009
20082008 = F
2008 2009

+F ( 2008 )
2008

1 1
= F
2008

2008
+ F 2008
2008

( 2008
)
1 1
= F + F 20082008 + F 2008
2008
(2008
)
1 1 1
= F
2008
+ F 2008 + F
2008
2007

(
+ F 2008
2008
)
1
= 2009 F
2008
(
+ F 2008
2008
)
1
( )
F 20082008 = 2008 F 2
= 2008 .
2008

II razred

1. Rjeenja nejednaine |z2| 2 ine zatvoreni krug. Neka je z = x+yi


dobijamo (x2)2+y2 2 koja definie zatvoreni krug sa centrom u taki
(2, 0) i poluprenika 2. Podijelimo sada ovaj krug pravima koje
prolaze kroz taku (2, 0) na 6 podudarnih zatvorenih krunih isjeaka,
Ik, k =0, 1, 2, 3, 4, 5, pri emu neka od tih pravih sadri neku taku recimo
z0 iz skupa S. Tada je rastojanje dvaju taaka sa proizvoljno odabranog
isjeka manje ili jednako od 2. Pretpostavimo da S ima vie od 5
taaka. Ako ima vie od 6 onda na osnovu Dirihleovog principa makar
dvije pripadaju nekom isjeku. Pretpostavimo da S ={zk, k = 0, 1, 2, 3, 4, 5}
ima tano 6 taaka. Pretpostavimo, odreenosti radi, da je zkIk,
k = 0, 1, 2, 3, 4, 5. Kako je broj z0 sa granice isjeka I0 on istovremeno
pripada isjeku I5. Znai |z0 z5| 2 to je trebalo dokazati.
2. Ako jednainu pomnoimo najmanjim zajednikim sadraocem dobijamo: (x - b)(x - c) + (x - a)(x - c) + (x - a)(x - b)
= 0, odnosno 3x2 - 2x(a + b + c) + (ab + bc + ac) = 0. Da bi rjeenja ove jednaine bila realna i razliita mora biti
diskriminanta D > 0. To dokazujemo ovako:
D = 4(a + b + c) 4 3(ab + bc + ca),
4(a + b +c) 12(ab + bc + ca) > 0 / : 2
2a + 2b + 2c 2ab 2bc 2ca > 0,
a - 2ab + b + a 2ac + c + b 2bc + c > 0,
(a b) + (a c) + (b c) > 0. Jasno je da posljednja nejednakost vai za sve razliite realne brojeve a, b i c
3. Neka su Ak, k = 1,..., 2008 tjemena mnogougla. Ako je EF diagonala
mnogougla onda tu nema ta dokazati. Pretpostavimo da je E taka dui
Ak Ak+1 i F taka dui Al Al+1. Vidjeti sliku: Al+1
Dokaimo da je EF max{EAk , EAk + 1}. Neka je, odreenosti radi, E
EAk + 1 EAk Tada je ugao FAk E FAk + 1E Kako je Ak FE spoljanji
ugao trougla FAk+1E, slijedi da je Ak FE >FA k+1 E. Zakljuujemo Al
da je FAk E<Ak FE, odnosno da u trouglu Ak FE vai sljedei
odnos stranica: Ak E > FE Ako sada na trouglu Ak Al Al+1 primjenimo
Ak
prethodni postupak dobijamo da je du Ak E manja od diagonale Ak Al F
ili diagonale Ak Al+1. Odavde zakljuujemo da je du EF manja od jedne Ak+1
dijagonale mnogougla

41
Takmienja
4. Traena povrina (vidi sliku) je P = PABCDE = 2(PAHE PGHD). AH = 2r, dok je stranica jednakostraninog trougla q =
r 3 Iz AFE je
AF = FE 3 (1) M
AF = AH FH = AH FE odnosno E
AF = 2r FE (2) D
2r
Iz (1) i (2) imamo FE 3= 2r - FE, tj. FE =
3 +1
=r ( )
3 1 .
A O F G H
a 3 r
Kako je AG = 3 = r , to je GH = = GD.
2 2 2
Uvrtavanje daje C

AH FE GH GD B
P = 2 = AH FE GH GD =
2 2 N
r r 9
( )
= 2r r 3 1 = r 2 2 3
2 2 4
L

Dakle P 1,21 r < /2r , te je traena povrina manja od polovine


povrine kruga.

III razred
1. Uslov an + 2 2an + 1 + an = 1 dat u zadatku moemo pisati kao
(an+2 an+1) + (an+1 an) = 1 (1)
Dakle, ako definiemo novi niz {bn} kao bn = an +1 an , n N , jednakost (1) postaje bn +1 bn = 1. Dakle, {bn} je
aritmetiki niz sa razlikom 1, odakle slijedi bn= b1+ n 1 = a2 a1+ n 1 za svaki prirodan broj n
Koristei prethodnu jednakost, dobijamo
an = (an an 1 ) + (an 1 an 2 ) + ... + (a2 a1 ) + a1 =
= bn 1 + bn 2 + ... + b1 + a1 =
1 a
= ( 2 1 + 2) + 1 =
n 2
b n a
( 1)( 2)
= ( 1) 1 + + 1 = ( 1)( 2 1 ) + 1
n 2n (n 1)(n 2)
n b a n a a + +a =
2
2 1
(n 1)(2n 2)
= (n 1)a (n 2)a +
(n 1)(n 2)
Dakle, za svaki prirodan broj n vai an = (n 1)a2 (n 2)a1 +
2
x+ y
2. Na osnovu nejednakosti xy , za x, y 0 (izmeu aritmetike i geometrijske sredine dva pozitivna broja)
dobijamo 2

a+b c+d
(a + b)(c + d ) = 2 2 4 ab 4 cd = 2 4 abcd
2 2
a+c b+d
(a + c)(b + d ) = 2
4 ac 4 bd 4
abcd
2 2
2 =2
a d b c
(a d )(b c) 4
ad 4 bc 4
abcd
+ +
+ + =2 2 =2
2 2
42
Takmienja
Sabiranjem prethodne tri nejednakosti dobijamo traenu nejednakost.

3. Kako je po uslovu zadatka A(x1, P(x1)) i B(x2, P(x2)), gdje je y1= P(x1), y2= P(x2) i x1 x2, to udaljenost izmeu taaka
A i B iznosi
2
P( x1 ) P( x2 )
| AB |= ( x1 x2 ) 2 + ( P ( x1 ) P ( x2 )) 2 =| x1 x2 | 1 + (1)
x1 x2
Stavimo P ( x) = an x n + an 1 x n 1 + an 2 x n 2 + ... + a1 x + a0 , pri emu su po pretpostavci an, an1, ... , a1, a0 cijeli brojevi
i an 0. Otuda imamo

P ( x1 ) P ( x2 ) = an ( x1n x2n ) + an 1 ( x1n 1 x2n 1 ) + ... + a1 ( x1 x2 ) (2)

Kako za svaki prirodan broj vai identitet ( x1k x2k ) = ( x1 x2 )( x1k 1 + x1k 2 x2 + ... + x1 x2k 2 + x2k 1 ) , zakljuujemo da je
cijeli broj x1k x2k djeljiv sa x1 x2
P ( x1 ) P ( x2 )
Koristei tu injenicu, na osnovu (2) slijedi da je m = cijeli broj, to uvrteno u (1) daje
x1 x2
| AB |=| x1 x2 | 1 + m 2
| AB |
Po uslovu zadatka |AB| je cijeli broj, pa stoga mora biti 1 + m2 = razlomak
| x1 x2 |
p
Stoga stavimo 1 + m2 = , gde su p i q relativno prosti prirodni brojevi.
q
Kvadriranjem prethodne jednakosti dobijamo (1 + m 2 )q 2 = p 2

Iz prethodne jednakosti vidimo da je p djeljivo sa q, pa zbog pretpostavke da su p i q relativno prosti prirodni


brojevi, zakljuujemo da mora biti q = 1 odnosno 1 + m2 = p2. Otuda je 1 = (p m)(p + m), to je jedino mogue u
sluaju p m = p + m = 1, tj. p = 1 i m = 1

P ( x1 ) P ( x2 )
Otuda slijedi = m = 0, odnosno P(x1) = P(x2), tj. y1 = y2, to je i trebalo dokazati.
x1 x2
4. Na osnovu slike imamo C
S ( ABC ) S ( A1B1C1 ) = S ( B1 AC1 ) + S (C1BA1 ) + S ( A1CB1 ) (1) A1
Nadalje vai: B1
1
S ( B1 AC1 ) = | AB1 | | AC1 | sin =
2
1
= | AB | cos | AC | cos sin = B
2 C1
A
1
= | AB | | AC | sin cos 2 =
2
= S ( ABC ) cos 2 .
Ovim smo dobili S ( B1 AC1 ) = S ( ABC ) cos 2 , a na analogan nain dobijamo
S (C1BA1 ) = S ( ABC ) cos 2 i S ( A1CB1 ) = S ( ABC )cos 2
Uvrtavanjem prethodna tri izraza u (1) dobijamo
S ( ABC ) S ( A1B1C1 ) = S ( ABC )(cos 2 + cos 2 + cos 2 )
Dijeljenjem gornje relacije sa S(ABC) dobijamo
S ( A1B1C1 )
1 = cos 2 + cos 2 + cos 2 , odakle slijedi traena jednakost
S ( ABC )

43
Takmienja
IV razred
1. Neka je O taka presjeka dijagonala kvadrata ABCD Trougao BDC1 je jednakokraki, pa je C1O njegova visina. Ako
je P presjek dijagonala kvadra, Q projekcija take P na ravan trougla BCD1. Taka P je podjednako udaljena od
tjemena B, D i C1, pa je Q centar opisanog kruga oko trougla BCD1. To znai da taka Q pripada dui C1O. Traeni
ugao je =PBQ
1
Oznaimo duinu ivice AB sa a i duinu ivice AA1 sa b Tada je PB = 2a 2 + b 2 . Ako je O1 presjek dijagonala
2
2
a 2 a 2 2
kvadrata A1B1C1D1, tada je C1O1 = , a primjenom Pitagorine teoreme C1O = + b
2 2
Iz slinosti trouglova POQ i C1OO1 dobijamo
b a 2 C1 B1

PQ PO ab
= , pa je PQ = 2 2 = .
C1O1 C1O a 2
2 a + 2b 2
2
+ b2 D1
2 A1
PQ ab
Tada je sin = = .
PB ( )(
a 2 + 2b 2 2a 2 + b 2 ) P
Q
Kako je traeni ugao otar, ugao e biti najvei kada je sin najvei.
Koristei transformacije dobijamo C B
ab O
sin = =
(a 2
)(
+ 2b 2a 2 + b 2
2
)
ab D A
= =
( 2a 4
+ 2b 4 + 5a 2b 2 ) C1 O1
1
= .
a 2 b2
2 2 + 2 + 5
b a
Primjenom nejednakosti artimetike i geometrijske sredine dobijamo da je
b P
a 2 b2 a 2 b2
2
+ 2 2 2 2 = 2, pa je Q
b a b a
1 1 1
sin = = .
a b 2 2 22 + 5 3
2 2 + 2 + 5
b a
C a 2 O
1 2
Otuda je najvea vrijednost traenog ugla jednaka arcsin .
3
2. Kako su funkcije f (t) = 2t i g (t) = t3 rastue vae sljedea tvrenja:
- ako je sin x > 0, tada je 2sin x > 20 = 1, pa je 2sin x 1 > 0,
- ako je sin x < 0, tada je 2sin x < 20 = 1, pa je 2sin x 1 < 0,
3
- ako je x2 > 0, tada je 2x 4 > 0,
3
- ako je x2 < 0 tada je 2x 4 < 0
Dakle oba sabirka su istog znaka, tj. istovremeno su oba pozitivna ili ova negativna. Jedina rjeenja jednaine
dobijaju se kada su oba sabirka jednaka nuli, tj. za x = 3 2 ili x = k , k Z .

3. Postavimo koordinatni sistem tako da se apscisa poklapa sa srednjom linjom mree, a ordinata da bude normalna na

44
Takmienja
apscisu i da bude osa simetrije mree. Neka je C tjeme pravog ugla, tada su na slici prikazane sve mogunosti:

1 2 3 4 5 6 7 8

Trouglova grupe 1, ije su obje katete paralelne koordinatnim osama, ima 3n 2 (n 1), jer tjeme C moe biti bilo
koji vor mree (3n mogunosti), dvije mogunosti za tjeme na vertikali i n 1 mogunost za tjeme na horizontali.
Trouglovi iz grupa 2, 3, 4, 5 i 6 su jednokraki sa katetama koji zaklapaju ugao od 45 sa koordinatnim osama.
Njih ima: 2 (n 2) (grupa 2, mnoimo sa 2 jer postoji simetrija u odnosu na apscisu, tjeme C ne moe biti u prvoj
i u 2008. koloni), 2 (n 2) + 2 (n 1) (grupa 3 i 4), 2 (n 4) (grupa 5) i 4 (n 3) (grupa 6, mnoimo sa 4 zbog
simetrije u odnosu na apscisu i ordinatu).

Trouglovi iz grupa 7 i 8 imaju katetu duine 5 . U grupi 7 ima ih 4 (n 3), a u grupi 8 ima ih 4 (n 5).

n
g (k ) n
g (k ) n
1
4. Za svaki prirodan broj k, vai k = 2 p ( k ) g (k ), pa je
k =1 k
= p(k )
k =1 2 g ( k )
= p(k ) .
k =1 2
n n n m
Meu brojevima 1, 2, ..., n postoji djeljivih sa 2, 2 djeljivih sa 22 ,... m djeljivih sa 2 , gdje je m
2 2 2
n n
prirodan broj takav da je 2m n, 2m+1 > n. Tada je brojeva za koje je p(k) = j jednak j j +1 , pa se traena
2 2
suma moe napisati na sljedei nain:

n
g (k ) m 1 n n

k =1 k
= j j j +1 =
j =0 2 2 2
m
1 n m 1 n
= j j
j 1 j =
j =0 2 2 j =1 2 2
n m 1 1 n
= 0 + j j 1 j =
2 j =1 2 2 2
m
1 n
=n j j
.
j =1 2 2
Kako je za svaki negativan realan broj x [x], to vai:

1
1 m
n
g (k ) m
1 n m
1 n m
1 1 4 1 1 2n n 2n

k =1 k
= n j j n j j = n n j = n n
j =1 2 2 j =1 2 2 j =1 4 4 1 1
= n n 1 m =
3 4 3 3 4
+ m
> .
3
4

45
Takmienja

Fizika
Zadaci
I razred
1. Mrav se udaljava od mravinjaka po pravoj liniji. Brzina mrava je proporcionalna kvadratnom korijenu njegovog
rastojanja od mravinjaka. U poetnom trenutku mrav se nalazio na rastojanju 1cm od mravinjaka i njegova brzina je
bila 2cm/s
a) Kolika e biti brzina mrava u trenutku kada se on nae na rastojanju 4cm od mravinjaka?
b) Nacrtati grafik zavisnosti reciprone vrijednosti brzine mrava od njegovog rastojanja od mravinjaka.
c) Na osnovu tog grafika, odrediti priblino u kom trenutku e rastojanje izmeu mrava i mravinjaka biti 4cm.
2. ovjek se nalazi u amcu na jezeru i eli da odredi masu amca M. Kako da to uradi, ako zna svoju masu m i ima
samo dugaku pantljiku (traku) kojom moe da izmjeri rastojanje od obale i duinu amca?
3. U Lamertovom eksperimentu (shema aparature kao na Sl. 1), odreuje se brzina molekula (atoma) gasa. U molekul-
skoj pei 2 dovodi se do eljene temperature, npr. gas atoma ive ili pare nekog drugog istopljenog metala. estice
gasa prolaze u snopu kroz niz pukotina (kolimator 4) u vakuumirani cilindar 1. U njemu se nalaze dva kruna diska,
5 i 6, sa uskim lunim prorezima irine = 2, koji su zakrenuti za ugao = 24 jedan u odnosu na drugi (kao na
Sl. 2). Rastojanje izmeu diskova je l = 40cm, a ugaona brzina kojom oni rotiraju oko ose cilindra je = 50 obrta/s.
estice gasa koje prou oba proreza, registruju se u detektoru 3. Odrediti maksimalnu i minimalnu brzinu estica
gasa
l
4
2 3
5 6

Sl 1 Sl 2
4. Ocijeniti srednju vrijednost gustine supstance Sunca. Poznat je period rotacije Zemlje oko Sunca T=1 god,
gravitaciona konstanta = 6,71011 Nm2/kg2 i ugao pod kojim se vidi prenik Sunevog kruga sa Zemlje
=2RS/r=0,01rad, gdje je RSpoluprenik Sunca, rrastojanje izmeu Zemlje i Sunca.
5. Satelit poluprenika Rsat se okree jednakom ugaonom brzinom oko svoje ose i oko masivne planete na rastojanju R
Izraunati kolinik momenta impulsa satelita L1za rotaciju oko planete i L2za rotaciju oko sopstvene ose. Poznat
je moment inercije kugle I = 2mR2/5

II razred

1. U izolovanom cilindrinom sudu nalazi se gas mase m i molarne mase M Gas je od


atmosfere odvojen klipom koji je za dno suda vezan oprugom konstante elastinosti k (slika
1). Na temperaturi T1 klip se nalazi na visini h od dna suda. Ako je trenje izmeu klipa i
suda zanemarivo, odrediti temperaturu T2 na koju treba zagrijati gas da bi se klip podigao na
visinu H Slika 1
2. Dva mola idealnog gasa na temperaturi T1 = 300K izohorno se hlade sve dok se pritisak ne smanji dva puta. Zatim
se gas izobarski iri dok mu temperatura ne postane jednaka poetnoj vrijednosti. Odrediti ukupnu koliinu toplote
koju je gas apsorbovao u ovim procesima. Univerzalna gasna konstanta je R = 8,314.

46
Takmienja

3. Na horizontalnoj podlozi lei pric prenika klipa D = 3cm i prenika otvora xd


d = 5mm (slika 2). Ako se klip kree konstantnom brzinom pod dejstvom F
konstantne sile F = 25N, odrediti domet mlaza vode iz prica xd Zanemariti
debljinu zida prica, gustina vode i gravitaciono ubrzanje su poznati.
Slika 2

4. Dvije paralelene metalne ploe, koje se nalaze na rastojanju d, povezane su


provodnikom. Izmeu njih se, paralelno njima, postavi trea metalna ploa
na koju je nanijeto naelektrisanje q (slika 3). Odrediti koliko naelektrisanje d
protekne kroz provodnik ako se trea ploa pomjeri paralelno samoj sebi za a
x, ostajui pri tome izmeu druge dvije ploe.
Slika 3

5. U kolu prikazanom na slici 4 otpor otpornika C iznosi R = 40, a izvor ima E


elekromotornu silu E = 2.5V i zanemariv unutranji otpor. Za otpornike
A
A i B ne vai Omov zakon U = IR, ve je veza izmeu struje i napona
data jednainom U = 2U2, gdje je 2 = 0.02A/V2 jednaka za oba otpornika. B
Odrediti:
a) Struju koja tee kroz otpornik A i
b) Vrijednost otpora R za koju e se na otpornicima B i C oslobaati jednake
snage C

Slika 4
III razred

B
1. Sistem prikazan na slici 1 sastoji se od gvozdene ipke mase m = 0,1kg i R2
elektrine otpornosti R1 = 0,1, koja klizi bez trenja po dvijema paralelnm R1 m
provodnim inama na meusobnom rastojanju l = 1m ine su nagnute u
odnosu na horizontalnu ravan pod uglom 30, a sa gornje strane su povezane
otpornikom R2 = 1. Magnetno polje indukcije B = 3T normalno je na M
ravan u kojoj lee ine. ipka je neistegljivim koncem povezana sistemom 30
koturova sa tijelom mase M = 1kg. Nakon nekog vremena tijelo se kree
konstantnom brzinom. Kolika je ova brzina? Slika 1
2. U elektromotorima uvijek ima kalemova sa velikim induktivitetom, koji
takoe imaju termogeni otpor. Veliki induktivitet uzrokuje i veliku faznu R
razliku izmeu jaine struje i napona, odnosno mali faktor snage. Da bi se
U C
poveao faktor snage, a time i aktivna snaga, paralelno se vezuje kondenzator
(slika 2). Ako su poznati kruna uestanost prikljuenog ulaznog napona L
(), induktivitet (L) i termogeni otpor (R) kalema, koliki kapacitet treba da
ima taj kondenzator da bi faktor snage bio maksimalan?
Slika 2
3. Tijelo miruje na strmoj ravni nagibnog ugla 3652' koja se nalazi na horizontalnoj podlozi. strma ravan pone
harmonijski da osciluje po horizontalnoj podlozi amplitudom 1mm. Koeficijent trenja izmeu tijela i strme ravni je
0,8. Pri kojoj minimalnoj frekvenciji oscilovanja tijelo poinje da klizi?
4. Longitudinalni talas prostire se kroz vazduh brzinom v0 = 340m/s. U nekom trenutku elongacije i brzine uoenih
estica sredine iznose x1 = 7105m, v1 = 4102m/s i x2 = 104m, v2 = 2102m/s Odrediti
a) krunu frekvenciju posmatranog mehanikog talasa,
b) amplitudu oscilovanja x0 estica pogoenih talasom,
c) rastojanje y izmeu ravnotenih poloaja ovih estica.
Promjena amplitude oscilovanja sa udaljenou od izvora se moe zanemariti.

47
Takmienja
5. Sistem prikazan na slici 3 sastoji se od sirene S mase m =1kg i tri opruge
istih koeficijenata elastinosti k = 100N/m. Sirena moe harmonijski da
osciluje po zakonu x = x0sin(t + ), gdje je poetna faza. U trenutku k k
t = 0 sirena je udaljena x1 = 5mm od svog ravnotenog poloaja i udaljava se
od njega brzinom intenziteta v1 = 7cm/s. Sirena tokom oscilovanja emituje S
zvuk nepoznate talasne duine 0 Prijemnik P koji miruje, registruje razliku
izmeu maksimalne i minimalne talasne duine zvuka koji do njega dopire. k
Smatrajui da nema interferencije, odredite nepoznatu talasnu duinu 0 P
Brzina zvuka u vazduhu je u = 330m/s
Slika 3
IV razred
1. Oblak od N = 104 pozitivnih piona kree se pod dejstvom magnetnog polja po krunici radijusa r = 20m brzinom
v = 0,99c. Pioni se raspadaju, po zakonu radioaktivnog raspada, i srednji ivot u njihovom sistemu im iznosi
= 26,1ns. Izraunajte:
a) Koliko piona ostane u oblaku posle jednog punog kruga?
b) Koliko bi piona ostalo nakon isteka istog vremena kad se oblak ne bi kretao? Dato je c = 3108 m/s
2. Zbog procesa na Suncu, najvjerovatnija talasna duina u spektru njegovog zraenja pomjera se sa 500 nm na 400 nm,
a poluprenik Sunca poveava se za 5%. Koliko e puta vie energije padati na Zemlju?
3. Nai maksimalnu kinetiku energiju elektrona izbaenih sa povrine litijuma elektromagnetnim zraenjem ije se
elektrino polje mijenja po Zakonu:
E= a (1 + cos(t)) cos(0t), gdje je
a konstanta,
=61014s1 i
0=361014s1
Izlazni rad za litijum iznosi 2,39eV. Poznata je konstanta h =6,621034Js.
4. Foton talasne duine i elektron kreu se du istog pravca jedan ka drugom. Pri sudaru, foton se raseje pod uglom od
180. Kolikom brzinom treba da se kree elektron prije sudara da se frekvencija fotona pri sudaru ne bi promijenila?
Date su potrebne konstante: h, c i me
5. Atom u kome oko protona, ne krui elektron, ve krui negativni mezon naziva se mezoatomom vodonika. Masa
mezona 207 puta je vea od mase elektrona, a naelektrisanja su im jednaka. Odredite vrijednosti rastojanja izmedju
protona i mezona i energije mezoatoma koristei Borovu teoriju. Proton i negativni mezon krue oko zajednikog
centra mase. Poznate su sljedee konstante:
h = 6,621034 Js,
e=1,61019 C,
me = 9,11031 kg i
k=9109 Nm2/C2

48
Takmienja

Rjeenja zadataka iz fizike


I razred

1. (a) Iz uslova zadatka je: = k x Odatle se dobija da je k = 1 x1 = 2 cm s

Onda je 2 = k x2 = 2 4 cm s = 4 cm s
(b)
1 s
v cm

0,55

0,5
1 1
0,45 =
v k x
0,4

0,35

0,3

0,25

1,5 2 2,5 3 3,5 4 4,5 x[cm]


1
(c) = x t , t = x Ptrapeza

a+b 1 1 1
Ptrapeza = h = ( + ) 3 = 9 / 8, t = 1,1s
2 2 2 4
2. Nalazei se na jednom kraju amca ovjek treba da ga postavi tako da on drugim krajem dodiruje obalu. Onda
treba da pree na drugi kraj amca kreui se konstantnom brzinom . amac e se pritom udaljavati od obale
konstantnom brzinom V.
Iz zakona odranja impulsa je:
m = MV (1)
Ako je t - vrijeme trajanja kretanja, l - duina amca, a x - rastojanje za koje se amac udalji od obale za vrijeme t,
onda je
t = l x (2) i
V t = x (3).
Iz (1),(2) i (3) slijedi da je m (l x) = M x , odakle se dobija M = m(l x 1) , l,x se mogu izmjeriti pantljikom.
3. Da bi estice gasa mogle dospjeti u detektor, mora biti ispunjen uslov
= t (1).
Vrijeme potrebno da estice preu rastojanje izmeu diskova je
t =l (2).
Odatle se dobija da je brzina estica
= l (3), tj. = 300 m/s
Zbog toga to prorezi imaju konanu lunu irinu , onda e u detektor ui pored estica brzine i sve estice, ije
su brzine u intervalu min = i max = +

49
Takmienja
Najsporija estica (min) je ona koja kroz prvi disk proe uz poetnu (lijevu) ivicu proreza, a kroz drugi disk proe uz
krajnju (desnu) ivicu proreza. Dok ona proe rastojanje izmeu diskova, oni se zakrenu za ugao +
Analogno prethodnom, najbra estica (max) je ona koja kroz prvi disk proe uz krajnju (desnu) ivicu proreza, a kroz
drugi disk uz prvu (lijevu) ivicu proreza. Dok ona proe rastojanje izmeu diskova, oni se zakrenu za ugao .
l l 2l
Slijedi da je: max min = 2 = = 2 ,
+ ( ) 2
l
tj = 2
= = 27, 3 m/s
1 ( ) 1 ( ) 2

II nain:
estica koja proe uz lijevi kraj proreza prvog diska moe proi uz lijevi
kraj drugog diska za vrijeme za koje se diskovi zarotiraju za ugao ili uz desni kraj drugog diska, za vrijeme za koje
se diskovi zarotiraju za ugao +
Razlika brzina estica je:
l l l
= = =
( + ) ( + ) +
Tako se dobija da je = 27, 3 m/s
Prema tome
min = = 272, 7 m/s i
max = + = 327, 3 m/s
4. Srednja gustina sustance Sunca je
MS M
S = = 3 S (1)
V 4 RS / 3
II Njutnov zakon:
mZ M S
mZ = mZ r 2 = (2)
r2
Na osnovu (1,2) slijedi:
r 3 2
MS = (3)

Po definiciji je:
2RS
= (4) i
r
2
= (5)
T
Postavljajui (3 i 5) u (1), dobija se:
(1,3)
3r 3 2 ( 5)
= = 24 3 2 (6)
4 RS3 T
Tako da se dobija na kraju 1,1 g/cm3
5. L1 = m R ,
RSAT
L2 = I , = / R ,
L1 m R mR 2
= = , tj R
L2 I / R I
2 2
L1 mR 5 R
= 2
= 2
.
L2 2mRsat / 5 2 Rsat
50
Takmienja

II razred
1. Na klip djeluju sila Zemljine tee m1g, sila pritiska gasa Fg = pS, sila pritiska
atmosfere Fa = p0S i sila elastinosti opruge Fe = kx, gdje su m1masa klipa, p
i p0pritisci u gasu i atmosferi, x deformacija opruge i Spovrina klipa. Ako Fg
pretpostavimo da je opruga istegnuta u odnosu na normalno, nedeformisano
stanje, poslednja sila ima smjer kao na slici 5.
Ako indeksima 1 i 2 oznaimo veliine vezane za temperature T1 i T2, jednaina
ravnotee na ovim temperaturama, redom moemo napisati u obliku: Fa Fc m1g
p0 S + kx1 + m1 g = p1S
p0 S + kx2 + m1 g = p2 S
Vodei rauna o tome da je promjena duine opruge jednaka promjeni visine
klipa, odnosno da je x2 x1 = H h , oduzimanjem prethodnih jednaina, lako
se moe dobiti:
S ( p2 p1 ) = k ( x2 x1 ) = k ( H h) (1)
slika 5
Jednaine stanja gasa na navedenim temperaturama mogu se napisati u obliku:
m
p1V1 = p1Sh = RT1
M
m
p2V2 = p2 SH = RT2
M
Iz ovih jednaina se moe dobiti:
mR T2 T1
p2 p1 = (2)
MS H h
Eliminacijom S iz jednaina (1) i (2) lako se moe odrediti traena temperatura do
koje treba zagrijati gas, koja iznosi:
H MkH
T2 = T1 + ( H h) p
h mR
1
2. p-V dijagram koji odgovara opisanom ciklusu prikazan je na slici 6. p1
Uvedimo sledee oznake za parametre stanja gasa:
T=
1: p1, V1, T1 Q1-2 co
ns
2: p2= p1/2, V2=V1, T2 t.
3: p3=p2=p1/2, V3, T1 3
p1/2
U izohoskom procesu 12 vai arlov zakon, pa je: 2 Q2-3
p1 p2 p T
= = 1 T2 = 1
T1 T2 2T2 2 V
Koliina toplote koju apsorbuje od okoline u ovom procesu iznosi:
1 V1 V3
Q1 2 = nCV (T2 T1 ) = nCV T1 slika 6
2
Znak minus pokazuje da u ovom procesu gas u stvari ne apsorbuje toplotu nego je predaje okolini. U izobarskom
procesu 23 gas apsorbuje od okoline koliinu toplote:
1
Q23 = nCP (T3 T2 ) = nCP (T1 T2 ) = nCPT1
2
Ukupna koliina toplote koju gas apsorbuje u ovim procesima jednaka je zbiru pomenutih toplota, odnosno:
1 1 1
Q = Q1 2 + Q23 = nCV T1 + nCPT1 = n(CP CV )T1
2 2 2
Ako se iskoristi Majerova relacija CP CV = R , moe se kazati da traena koliina toplote iznosi:
1
Q = nRT1 2490J = 2.49kJ
2

51
Takmienja
3. Pritisak u vodi neposredno uz klip (presjek 1 strujne cijevi) jednak je zbiru (1)
atmosferskog pritiska i pritiska sile F, a na otvoru (presjek 2) jednak je (2)
F v1 v2
atmosferskom pritisku (slika 7).
pat pat
Poto je strujna cijev horizontalna, Bernulijeva jednaina primijenjena na
ove presjeke moe se pisati u obliku:
xd
v12 F v2
+ pat + = 2 + pat
2 S1 2
slika 7
Ako se na iste presjeke primijeni jednaina kontinuiteta i transformie saglasno geometrijskim odnosima, moe se
Sv D2
dobiti: v1 = 2 2 = 2 v2
S1 d
Posle uvrtavanja ove brzine u Bernulijevu jednainu moe se izraziti brzina mlaza vode na izlazu iz prica, koja
2F
iznosi: v2 = 2 D
( D 4 d 4 )
estice mlaza posle izlaska iz prica kreu se po zakonima kretanja horizontalnog hica, iji domet iznosi:
2h
xd = v2
g
I oigledno zavisi od visine h sa koje polazi. Najvei domet imaju djelovi mlaza koji izlaze kroz najvii dio otvora.
Primijetimo da prenik otvora prica nije mnogo manji od prenika prica (D / d = 6 : 1), pa je maksimalan domet
mogue odrediti ako se uzme h = ( D + d ) / 2. Meutim,
D 4 >> d 4 pa je:
2 2F
v2
d
a traeni domet iznosi:
2 2F (D + d )
xd 0.503m 50.3cm
d g

4. Oznaimo indukovana naelektrisanja na ploama sa q1 i q2 (slika 8). ? ? ? ?


q1
Prikazani sistem se moe posmatrati kao sistem od dva kondenzatora.
Oigledno je: + + + + + + + +
+q1
+ + + + + + + + + + + + + + + + +
+q2
q1 + q2 = q (1)
q2
? ? ?
?
Treba primijetiti da se na spoljanje povrine spoljanjih ploa rasporeuje q1 +q1
koliina naelektrisanja , da bi ploe kao cjelina ostale neutralne kao i
prije unoenja tree ploe u prostor izmeu njih. Ova naelektrisanja se +q

rasporeuju tako da bi pomenute ploe bile na istom potencijalu. Raspored q2 +q2


ovih naelektrisanja nam nije vazan, pa su na slici oznaena upitnicima.
slika 8
Poto su spoljanje ploe spojene provodnikom, a nalze se na istom potencijalu, pa je potencijalna razlika izmeu
unutranje (zajednike) i spoljanjih ploa ovih kondenzatora jednaka. Iz jednakosti napona na kondenzatorima
slijedi:
q1 q2 q1 q2
= , = odnosno:
C1 C2 S S
0 0
a d a
q1a = q2 (d a ) (2)
Iz jednaina (1) i (2) lako se mogu odrediti naelektrisanja na kondenzatorima koja iznose:
a d a
q1 = q i q2 = q
d d

52
Takmienja
Kada se trea pomjeri za x, na primjer ka donjoj ploi, na isti nain se mogu dobiti nova naelektrisanja na ploama
kondenzatora, koja iznose:
a+x d ax
q '1 = q i q '2 = q
d d
Pri navedenom pomjeranju tree ploe, kroz provodnik protekne naelektrisanje:
x
q = q1 q '1 = q2 q '2 = q
d
Lako se moe pokazati da bi pomjeranje za isto rastojanje tree ploe ka gornjoj ploi dovelo do istog rezutata.
5. Prema postavci zadatka, ako kroz otpornike A i B protie struja I, na njegovim krajevima se javlja napon U = I / .
Ako pretpostavimo smjerove struja kao na slici,I Kirhofovo pravilo za vorove moe biti napisano u obliku:

I A = I B + IC (1)
E
A II Kirhofovo pravilo za odgovarajue konture u obliku: IA
A
I I
EAB: E = A + B , (2)
B

IB
BC: 0 = RI C (3) IB
IC
a) Iz poslednje dvije jednaine lako se moe dobiti:
C
I B = R 2 2 I C2 (4)
I A = 2 ( E RI C ) 2 Slika 9
Uvrtavanjem ovih vrijednosti u jednainu (1), moe se odrediti struja kroz otpornik C, koja iznosi:
2E2
IC = ,
1 + 2 RE
pa traena struja kroz otpornik A iznosi:
1 + 2 RE
I A = 2E2 2 = 0.045 A = 45mA
1 + 2 RE
b) Poto su otpornici B i C vezani paralelno, naponi na njima su jednaki, pa se na njima oslobaaju jednake snage
ukoliko kroz njih teku iste struje, odnosno, ako je I B = I C . Ako iskoristimo ovu jednakost u jednaini (4), lako se
moe dobiti da je u tom sluaju:
1
I B = IC = 2 2
R
a ako isto iskoristimo u jednaini (1), moe se dobiti da je tada:
I A = 2I B
Ako se ove vrijednosti uvrste u jednainu (2), moemo dobiti traenu otpornost otpornika A, koja iznosi:
2 +1
R = 2 = 48.3
E

III razred
1. Kada se pusti tijelo mase M, ono se sputa ubrzano. Kako je ovaj teg neistegljivim koncem povezan sa ipkom
i ipka e se kretati ubrzano. Zbog kretanja ipke doi e do pojave indukovane struje, tako da e na provodnik
djelovati i magnetno polje. Ubrzanje sistema e se smanjivati i kada ono bude jednako nuli tijelo M e se kretati

stalnom brzinom. Na slici r1 oznaene su sile koje djeluju u sistemu. Sile F/ / , T , FA , djeluju na ipku. Kako se ipka
kree stalnom brzinom vm, rezultanta ovih sila mora biti jednaka nuli, pa je
IBl = mg sin + T (1)
Na tijelo djeluju sile 2T i Mg. njihova rezulatanta je takoe jednaka nuli, pa je 2T = Mg, tj
T = Mg / 2 (2)

53
Takmienja
Struja koja se indukuje zbog kretanja provodnika je B
i Blvm
I= = (3) R2
R1 + R2 R1 + R2 Fa
Fr T
Zamjenom (2) i (3) u (1) i sreivanjem dobija se T
30
R1 + R2 M m T
vm = g m sin + = 0, 66 T Mg
B 2l 2 2 s
T
Kako je brzina tijela vM = vm/2, za brzinu tijela se dobija
mg
R +R M m
vM = 1 2 22 g m sin + = 0, 33 Slika r1 I1
2B l 2 s I
2. Maksimalni faktor snage dobija se za sluaj kad je fazna razlika izmeu struje i napona u kolu
jednaka nuli. na slici r2 prikazan je odgovarajui vektorski dijagram. Sa slike se vidi da je: R

U U C I1 I2
I1 = I 2 sin ili U C = sin L
R 2 + ( L ) 2 U

je fazni pomjeraj izmeu struje kroz granu sa kalemom i napona na krajevima te grane, pa je
I
UL L U L 1
UL
sin = = U C = I
2 2 2 2 2 2 2 2
UL +UR ( L ) + R R + ( L ) R + ( L )
Iz prethodnih izraza slijedi da je, R
U=UC
I
L U C I 1
I 2
C= 2 2 L
R + ( L ) UR
I2 Slika r2
3. Kada strma ravan miruje, tijelo na njoj takoe miruje i intenzitet
maksimalne sile statikog trenja je vei od komponente sile zemljine
tee koja bi pomjerala tijelo niz strmu ravan.
Ako strma ravan osciluje, na tijelo djeluje inercijalna sila koja je
Fi1
periodina. Amplitudna vrijednost te sile je Ftr
Fi
Fi 0 = mai 0 = mx0 2
Da bi tijelo poelo da klizi, potrebno je da komponenta inercijalne sile
koja je paralelna sa strmom ravni ispunjava uslov (slika r3), F in Fn a
Fi 0|| Ft F||
gdje je Ft = (mg cos Fi 0 sin )
U graninom sluaju je mg F1
2 2
mx0 cos = m( g cos x0 sin ) mg sin . Slika r3
Sreujui prethodnu jednainu i stavljajui = 2 min dobija se
1 g ( cos sin )
min = = 2, 8Hz
2 x0 (cos + sin )
4. a) U trenutku t elongacije su date sa x1 = x0 cos(t ky1 ) i x2 = x0 cos(t ky2 ), a odgovarajue brzine
v1 = x0 sin(t ky1 ) i v2 = x0 sin(t ky2 ). Kvadriranjem i oduzimanjem prethodnih jednaina dobija se
v12 v22
v12 v22 = 2 ( x22 x12 ) = = 485rad/s
x22 x12
b) Kvadriranjem i sabiranjem izraza za x1 i v1, ili za x2 i v2, uz korienje osnovnog trigonometrijskog identiteta
dobijamo
x0 = x12 + v12 / 2 = x22 + v22 / 2 = 1, 08 104 m
c) Faze oscilovanja uoenih taaka su
t ky1 = arccos( x1 / x0 ) i t ky2 = arccos( x2 / x0 )
Oduzimanjem prethodnih jednaina dobijamo
y = y2 y1 = 1 / k (arccos( x1 / x0 ) arccos( x2 / x0 ))
54
Takmienja
Poto je k = 2 / i v0 = , onda je k = / v0 , pa dobijamo y = 0, 335m
5. Minimalana talasna duina zvuka na prijemniku je
u vmax u x0
min = 0 = 0 , a maksimalna talasna duina iznosi
u u
u + vmax u + x0 u
max = 0 = 0 . Odavde je 0 =
u u 2 x0
Kako je ekvivalentni koeficijent elastinosti 3k, slijedi da je 2 = 3k / m
Iz x1 = x0 sin i v1 = x0 cos , slijedi tan = x1 / v1
tan
Koristei relaciju sin = , za amplitudu oscilovanja dobijamo
1 + tan 2
u u
x0 = v12 + x12 2 / , pa je 0 = = 1, 5m
2 v12 + x12 2 2 v12 + 3kx12 / m

IV razred
2 r
1. a) U laboratorijskom sistemu proe vrijeme = = 423, 2ns.
v
v2
U sistemu piona tom vremenu odgovara vremenski interval t ' = t 1 = 59, 69ns.
t'
c2
N
N = N 0e
= 0,102 = 10, 2%. Nakon jednog okreta ostaje N=1020 piona.
N0 t
N
b) Kad bi u sistemu piona proao isti period vremena kao u laboratorijskom sistemu, imali bismo = e = 9,12 108.
Piona dakle, nebi ni bilo. N0
2. Na poetku procesa snaga zraenja koja pada na Zemlju je:
S R2 ' R '2
1 = = T14 S2 r 2 , a na kraju 2 = S = T24 2S r 2
4 R 4 R
4 2
1 T2 R 'S
Deobom se dobija: = .
2 T1 RS
4 2
1, 05 RS
Kako je: 1 T1 = 2 T2 = b , slijedi: 1 = 2 = 2, 7.
2 1 RS
3. Zakon po kome se mijenja elektrino polje datog zraenja moe se napisati u obliku:
1 1
E = a cos(0t ) + a cos(( + 0 )t ) + a cos((0 )t ).
2 2
U tom zraenju su zastupljeni fotoni frekvencija:
0
1 = = 5, 73 1014 Hz
2
0 +
2 = = 6, 68 1014 Hz
2
0
3 = = 4, 77 1014 Hz
2
A
Najmanja frekvencija zraenja koje moe izazvati fotoefekat sa povrine litijuma je 0 = i = 5, 77 1014 Hz.
h
Fotoefekat mogu izazvati samo fotoni frekvencije 2. Tada je maksimalna kinetika energija fotoelektrona:
T = h Ai = 0, 37eV .

55
Takmienja
4. Po zakonu odranja energije h + Te = h '+ Te ' , a kako je, po uslovu zadatka = ' , slijedi Te ' = Te , tj. Intenzitet
brzine se ne mijenja, ali se po zakonu odranja impulsa, mijenja njen smjer:
h h ' h h h mv h
pe = pe ' = pe Slijedi pe = = ; =
c c c c v2
1
c2
c
Odatle se nalazi v = , gdje je E0 energija mirovanja elektrona.
2
E
1+ 0
hc
5. U mezoatomu estice krue oko zajednikog centra mase uslijed privlane elektrostatike interakcije. Rastojanja
estica od centra mase su r1 i r2, pa vai relacija: m1r1 = m2 r2 , a rastojanje izmeu estica je: d = r1 + r2 . Slijedi:
m2 d m1d
r1 = ; r2 =
m1 + m2 m1 + m2
Jednaine kruenja estica oko centra mase su:
v12 e2
m1 =k 2
r1 d
odnosno
v22 e2
m2 =k 2
r2 d
m1 k e2
(m1 + m2 ) v12 =
m2 d
(1)
m2 k e2
(m1 + m2 ) v22 =
m1 d
Po Borovom postulatu kvantovanja momenta impulsa vai jednaina:
h
L = m1 v1 r1 + m2 v2 r2 = n , n = 1, 2, 3....
2
Odatle slijedi:
h
v1 + v2 = ,
2 m1 r1
a kako je:
v1 v2 r h
= , to je: v1 1 + 2 = n ; slijedi
r1 r2 r1 2 m1 r1
h h
v1 = n = n ;
2 m1 (r1 + r2 ) 2 m1 d
h
slino tome: v2 = n ; (2)
2 m2 d
(m1 + m2 ) h 2
Iz jednaina (1) i (2) dobija se: d = n 2 = n 2 2, 84 1013 : m, n = 1, 2, 3...
4 2 m1 m2 k e 2
Ukupna energija mezoatoma vodonika je:
1 1 k e2
E= m1 v12 + m2 v22
2 2 d
1 2 2 k 2 e4
= 2
n h2
1
= 2 2, 54keV , n = 1, 2, 3,...
n
56
Takmienja

Informatika
Zadaci
IV razred
1. Jedan prozor se sastoji od ivice koju prikazujemo simbolima '-' (minus), '|' (vertikalna crta) i '+' (plus), od unutranjosti
prozora koja je popunjena znakovima '.' (taka), a na sredini gornje ivice prozora nalazi se naziv prozora izmeu
dvije vertikalne crte. Naziv e biti centriran ili malo lijevo ako ga je nemogue tono centrirati. Rastojanje izmeu
prvog slova naziva i lijeve ivice prozora bie jednaka ili za tano jedan manje od rastojanja posljednjeg slova naziva
od desne ivice prozora. Drugim rijeima, naziv e biti centriran koliko je to mogue. Naziv prozora sastoji se od
malih slova engleske abecede. Svaki prozor je dovoljno irok da cio naziv stane na gornju ivicu prozora zajedno sa
susjednim znakovima '-|' slijeva tj. '|-' zdesna. irina prozora bie barem za 6 kolona vea od duine naziva. Visina
svakog prozora bie najmanje tri reda, a nee postojati dva prozora s istim nazivom.
Dat je izgled ekrana s vie prozora, tako da se nijedna dva prozora ne prekrivaju (ni djelomino ni potpuno).
Prozore treba sloiti u takozvani "cascade mode" na sljedei nain:
visina i irina svih prozora mora ostati nepromijenjena
gornja lijeva ivica prvog prozora treba biti u gornjem lijevom uglu ekrana
svaki sljedei prozor treba prekrivati prethodni i biti pomaknut tono za jedan red dolje i za jednu kolonu udesno
prozori trebaju biti poredani abecedno po nazivu prozor iji je naziv prvi po abecedi ide u prvi red, drugi po
abecedi u drugi red itd
Ulazni podaci
U prvom nalaze se prirodni brojevi M i N, 100,10NM broj redova i broj kolona ekrana.
U svakom od sljedeih M redova nalazi se po N karaktera koji predstavljaju izgled ekrana.
Naziv svakog prozora sastojae se od najmanje jednog a najvie 10 malih slova engleske abecede. Brojevi M i N bie
dovoljno veliki da svi prozori poslije slaganja stanu na ekran.
Izlazni podaci
U svaki od M redova potrebno je tampati po N karaketra tj. traeni izgled ekrana nakon slaganja prozora.
Primjeri podataka za testiranje

ulaz ulaz ulaz


12 12 15 22 14 15
............ ...............+-|a|-+ +---|a|---+....
.+-|ana|-+.. .+-|winamp|--+.|.....| |.........|....
.|.......|.. .|...........|.|.....| |.........|....
.|.......|.. .|...........|.|.....| +---------+....
.|.......|.. .|...........|.|.....| ...............
.+-------+.. .|...........|.|.....| ...............
..+-|bozo|-+ .+-----------+.|.....| +--|ab|---+....
..|........| ...............|.....| |.........|....
..|........| +----|kit|----+|.....| |.........|....
..|........| |.............||.....| +---------+....
..|........| |.............||.....| +--|abc|--+....
..+--------+ |.............||.....| |.........|....
|.............|+-----+ |.........|....
izlaz |.............|....... +---------+....
+-|ana|-+... +-------------+.......
|+-|bozo|-+. izlaz
||........|. izlaz +---|a|---+....
||........|. +-|a|-+............... |+--|ab|---+...
+|........|. |+----|kit|----+...... ||+--|abc|--+..
.|........|. ||+-|winamp|--+|...... +||.........|..
.+--------+. |||...........||...... .+|.........|..
............ |||...........||...... ..+---------+..
............ |||...........||...... ...............
............ |||...........||...... ...............
............ |++-----------++...... ...............
............ |.....|............... ...............
|.....|............... ...............
|.....|............... ...............
|.....|............... ...............
+-----+............... ...............
......................
......................

57
Takmienja
2. U vatrogasnu jedinicu stiglo je saoptenje o moguem poaru u jednom sektoru guste ume oblika kvadrata. Da bi
pronali mjesto poara, deurni je pozvao vazdunu slubu osmatranja i poslao N aviona da pretrae dati sektor I
ustanove tano mjesto poara. Nijedan od aviona nije prijavio poar. Poznato je da se avion kree pravolinijski i pilot
vidi pojas ume irine 50km lijevo i desno od linije na povrini zemlje nad kojom se kree avion (vidi sliku), gdje su
take koje su udaljene tano 50km od linije vidljive iz aviona.
Po povratku u bazu, svaki od pilota je prijavio taku (xb,yb) u kojoj je uao u sektor ume i taku (xe,ye) u kojoj je
napustio sektor. Izmeu te dvije take avion se kretao pravolinijski.
Treba napsiati program koji na osnovu izvjetaja pilota odreuje da li je stvarno itav kvadrat ume bio pregledan.
Ako nije, treba nai koordinatu bar jedne take u kvadratu ili njegovim stranicama koja nije obuhvaena pregledom
iz aviona.

Ulazni podaci
Ulazna datoteka ima N+2 reda. U prvom redu upisana je duina L stranice kvadrata u kilometrima, (0 < L 1000).
U drugom redu ulaza je prirodan broj N broj aviona koji pretrauju umu, 1 N 100. U svakom od sljedeih N
redova upisana su po etiri realna broja koordinate xb, yb, xe, ye, razdvojene jednom bjelinom. Koordinate su zadate
u kilometrima. Donji lijevi ugao kvadrata ima koordinate (0, 0) a gornji desni ugao kvadrata ima koordinate (L. L).

Izlazni podaci
Izlazna da sati u kilometrima, sa tanou od 1 metra.

Primjeri test podataka


ulaz
245 1 26.1 0 193.568 245

izlaz
155.123 100

3. Na krunoj autobuskoj liniji duine S kree se N autobusa koji su oznaeni brojevima 1, 2, ..., N, po redosljedu
polazaka. Autobus broj 1 kree se iza autobusa broj N. Autobusi se kreu istom brzinom V0 i sa jednakim vremenskim
intervalima izmeu autobusa. Dispeer u podne istovremeno uklanja sa linije K autobusa i alje vozae tih autobusa
na ruak. Da bi se vratili jednaki intervali kretanja izmeu autobusa potrebno je neko vrijeme T i, moda, promjena
brzine kretanja nekih autobusa koji su ostali na liniji. Za vrijeme tog perioda, autobusi se moraju kretati konstantnim
brzinama iz intervala [Vmin, Vmax], pri emu brzine zadaje dispeer. Promjena brzine izvodi se trenutno. Poslije isteka
vremena T, autobusi ponovo nastavljaju da se kreu brzinom V0
Va zadatak je da napiete program za automatskog dispeera, koji izraunava minimalno vrijeme Tmin za koje
intervali izmeu autobusa postaju jednaki i brzinu kretanja svakog od autobusa.

Ulazni podaci
U prvom redu ulazne datoteke nalaze se prirodni brojevi N, K, S, Vmin, Vmax i V0, gdje je K<N10000, S10000,
Vmin < Vmax 10000, Vmin V0 Vmax). U drugom redu su u rastuem poretku dati K brojeva autobusa koji se uklanjaju
sa linije u podne. Svi podaci u razdvojeni jednom bjelinom.

Izlazni podaci
U prvom redu izlazne datoteke treba upisati vrijeme Tmin. U svakom od sljedeih N-K redova upisati po dva broja
razdvojena bjelinom: broj autobusa i brzina kretanja autobusa u toku inetrvala Tmin

Primjeri test podataka


ulaz ulaz
4 1 60 21 70 60 3 4 2 40 30 80 50 2 4

izlaz izlaz
0.2041 1 45.5 2 21 4 70 0 1 50 3 50

58
Takmienja

Rjeenja zadataka iz informatike


IV razred
1. Opis algoritma:
Program se odvija u sljedecim koracima:
1. Ucitamo sve ulazne podatke u matricu.
2. Nadjemo sve prozore, odredimo im sirinu, visinu i naslov. Usput brisemo sve prozore iz matrice (dovoljno je
brisati samo ivice prozora, jer je unutrasnjost prazna). Sad je matrica prazna (sadrzi samo tacke).
3. Sortiramo sve prozore po naslovu (bilo koji algoritam sortiranja je dobar)
4. Crtamo sve prozore u matricu, tako sto crtamo i unutrasnjost proz, jer na taj nacin brisemo dejlove prozora koji
su ispod
5. Stampamo sadrzaj matrice.
Slozenost rjesenja je O(N3)

program kaskade;
const MAXN = 100;
MAXM = 100;
MAXP = 100;
type Tnaslov = string[10];
type Tprozor = record
naslov : Tnaslov;
sir, vis : longint;
end;

var a : array[1..MAXN+MAXN, 1..MAXM+MAXM] of char;


p : array[1..MAXP] of Tprozor;
temp : Tprozor;
n, m, np, i, j, k : longint;

begin
(* ucitavanje... *)
readln(n, m);
for i:=1 to n do
begin
for j:=1 to m do read(a[i, j]);
readln;
end;
(* ...ucitavanje *)

(* trazenje prozora... *)
np := 0;
for i:=1 to n do
for j:=1 to m do if a[i, j]='+' then
begin
np:=np+1;
(* trazimo sirinu... *)
for k:=j+1 to m do if a[i, k]='+' then
begin
p[np].sir := k-j+1;
break;
end;
(* trazimo visinu... *)
for k:=i+1 to n do if a[k, j]='+' then
begin
p[np].vis := k-i+1;
break;
end;

(* spremamo naslov... *)
p[np].naslov := '';
k:=j+1;
while a[i, k]<>'|' do k:=k+1;
k:=k+1;
while a[i, k]<>'|' do

59
Takmienja
begin
p[np].naslov := p[np].naslov + a[i, k];
k:=k+1;
end;
(* jos preostaje prebrisati prozor... *)
for k:=1 to p[np].sir do
begin
a[i, j+k-1] := '.';
a[i+p[np].vis-1, j+k-1] := '.';
end;
for k:=1 to p[np].vis do
begin
a[i+k-1, j] := '.';
a[i+k-1, j+p[np].sir-1] := '.';
end;
(* ...prozor je prebrisan *)
end;
(* ...trazenje prozora *)
(* sortiranje prozora... *)
for i:=1 to np do
for j:=i+1 to np do
if p[i].naslov>p[j].naslov then
begin
temp := p[i];
p[i] := p[j];
p[j] := temp;
end;
(* ...sortiranje prozora *)

(* crtanje prozora... *)
for k:=1 to np do
begin
(* vodoravne crtice... *)
for i:=1 to p[k].sir-2 do
begin
a[k, k+i]:='-';
a[k+p[k].vis-1, k+i]:='-';
end;
(* naslov... *)
j:=(p[k].sir-length(p[k].naslov)) div 2;
a[k, k+j-1]:='|';
a[k, k+j+length(p[k].naslov)]:='|';
for i:=1 to length(p[k].naslov) do a[k, k+j-1+i]:=p[k].naslov[i];

(* vertikalne crtice... *)
for i:=1 to p[k].vis-2 do
begin
a[k+i, k]:='|';
a[k+i, k+p[k].sir-1]:='|';
end;

(* kutovi... *)
a[k, k]:='+';
a[k, k+p[k].sir-1]:='+';
a[k+p[k].vis-1, k]:='+';
a[k+p[k].vis-1, k+p[k].sir-1]:='+';

(* sredina... *)
for i:=1 to p[k].vis-2 do
for j:=1 to p[k].sir-2 do a[k+i, k+j]:='.';
end;
(* ...crtanje prozora *)
(* stampanje podataka... *)
for i:=1 to n do
begin
for j:=1 to m do write(a[i, j]);
writeln;
end;
(* ...stampanje podataka *)
end.

60
Takmienja
2. Prvo rjeenje: Posmatrajmo sljedei skup dui: 4 stranice kvadrata i 2N dui koji su presjek pojasa koji su avioni
pregledali sa stranicama kvadrata. Odredimo sve presjeke tih dui, po parovima. Ako postoji taka koja nije
pregledana, ona mora biti u okolini neke od presjenih taaka posmatranih dui. Razmotrimo 4 ugla koja se dobijaju
kao presjek dva pojasa koja su pregledala dva aviona i neka je
A tjeme takvog ugla (vidi sliku). Ako je jedna od dui stranica
B2
kvarata, treba posmatrati samo uglove koji su u unutranjsoti
kvadrata. Odredimo presjeke simetrala svakog od uglova sa
najbliom dui koja ne prolazi kroz taku A i oznaimo te B4
take sa B1, B2, B3 i B4. Dovoljno je provjeriti da li sredita A
B3
dui AB1, AB2, AB3 i AB4 pripadaju nekom od pojaseva.
Drugo rjeenje: Provjerimo da li sva 4 tjemena kvadrata
lee u nekom od pojaseva ili postoji jedno od tjemena koje
nije pregledano nijednim od aviona. U oba sluaja, zadatak je B1
rijeen. Ako zadatak nije rijeen, podijelimo svaku stranu na
polovine i dobijemo 4 manja kvadrata. Analiziramo tjemena
svakog od dobijenih manjih kvadrata. Sve kvadrate ija su sva
4 tjemena pokrivena jednim pojasom eliminiemo iza razmatranja. Ovim postupkom naiemo rjeenje ili e duina
stranice kvadrata postati toliko mala da e raunar smatrati da je duine 0. U drugom sluaju, kvadrat prelazi u taku
i smatraemo da je pregledan nekim od aviona.
3. Autobusi idu redom jedan iza drugog: broj 2 iza broja 1, broj broj 3 iza broja 2, ..., broj 1 iza broja N. Autobus koji za
vrijeme Tmin prelazi najdui mogui put mora se kretati najveom brzinom Vmax. Ako odredimo najdui put i znajui
Vmax, moe se odrediti Tmin. Znajui vrijeme i preena rastojanja drugih autobusa, lako je odrediti njihove brzine.
Prvo odredimo rastojanja izmeu autobusa poslije uklanjanja K autobusa sa linije. Zajedno sa tim dijelom koda
pamtimo i brojeve preostalih autobusa.
J:= N - K + 1;
for I:= N downto 1 do
if M[I] = 0 then inc(MDist[J])
else begin dec(J); MDist[J]:= 1; MNum[J]:= I; end;
MDist[1]:= MDist[1] + MDist[N - K + 1];
Rastojanje izmeu preostalih autobusa raunamo kao broj intervala izmeu autobusa prije uklanjanja K autobusa
sa linije, pa koristimo niz cijelih brijeva a ne niz realnih brojeva. Polazimo od toga da je brzina prvog autobusa
minimalna, i raunamo puteve koji su preli ostali autobusi, pamtei najdui i najkrai put.
MaxWeg:= 0; MinWeg:= 0; D:= 0; Flag:= false; T:= 0; VVV:= V0;
for I:= 2 to N - K do begin
Work:= MDist[I] * FInt - LInt + D;
if abs(Work) < 5.E-10 then Work:= 0;
if Work <> MaxWeg then Flag:= true;
if Work > MaxWeg then MaxWeg:= Work;
if Work < MinWeg then MinWeg:= Work;
D:= Work;
end;
Sada odreujemo vrijeme prvog autobusa
if Flag then begin
T:= (MaxWeg - MinWeg)/(VMax - VMin); VVV:= -MinWeg/T + VMin;
end;
writeln(Out, T:1:4); writeln(Out, MNum[1], ' ', VVV:1:4);
I na kraju:
D:= 0;
for I:= 2 to N - K do begin
Work:= MDist[I] * FInt - LInt + D;
if Flag then VVV:= (Work - MinWeg)/T + VMin;
writeln(Out, MNum[I], ' ', VVV:1:4);
D:=Work;
end;

61
U koraku sa...

Kako uiti informatiku


Neto Vam nije jasno? Nikada niste uli za to? Onda ste osjetili poentu ! ! !

U srednje kole Crne Gore nedavno je uveden novi nastavni funkcija je da automatski generie hipertekst na osnovu
program po kome se izuava svijet moderne informatike. podataka koje mu unese korisnik. Da bi se iskoristile sve
U sklopu opte gimnazije informatika je uvedena kao mogunosti ovog jezika, esto je potrebno poznavati i SQL
obavezan izborni predmet za prvu godinu i omoguena je baze podataka kao i njihov interpretator MySQL. Njegovo
kao izborni za drugu, treu i etvrtu godinu. korienje omoguava PHP-u da sauva i ponovo koristi
Dakle, u prvoj godini ue se dva najosnovnija programa podatke koje korisnik unese. Ovako funkcioniu blogovi
iz Microsoft office paketa, Word i Power Point. Ova dva i forumi
osnovna programa iskoriena su za upoznavanje uenika Pored poglavlja o programiranju (Web development),
sa interfejsom, kreiranjem datoteka i slino. Problem razvijeno je i ono o dizajniranju Web stranica. Dananji
se javlja kada se u drugoj godini prelazi na naprednije Web dizajneri koriste alate kao sto su Photoshop, za obradu
tehnologije. Microsoft office sve vie profesionalaca smatra fotografija, ili Flash, za izradu animacije, koje esto i same
neozbiljnim kada se radi o Web dizajnu (Ekspression Web) imaju mogunost interaktivnosti sa korisnikom.
i radu sa bazama podataka (Access). Uprkos ovome, ova Prema ovome za uenike koji izaberu da se bave
dva programa ulaze u predmetne programe namijenjene informatikom bilo bi korisno uvesti programe i tehnologije
drugoj i treoj godini. Idemo li naprijed ili nazad? koji su upotrebljiviji i moderniji u svijetu informatike. Evo
Za uenike druge godine predvieno je da izuavaju Web jednog predloga:
prezentacije, tj. dvije osnovne tehnologije koje pokreu Od kako su se prole godine ujedinili Adobe i Macromedia,
statine Web stranice: CSS i XHTML kroz upotrebu ve novi paket Creative Suite 3 sadri sve to jednom
pomenutog Expression Web-a. dizajaneru, Web i grafikom, animatoru, video ili audio
CSS (Cascading Style Sheets) namijenjen je izgradnji stila producentu moe zatrebati a cijena nije nita vea od
Web stranice, tanije, koristi se za podjelu Web sajta po Microsoft-ovih paketa.
tabelama i dodjeljivanje karakteristika (pozadinska boja Naravno, bilo bi previe traiti od uenika srednje kole
poznavanje svake od ovih oblasti.
Osnovnim paketom moglo bi se
omoguiti:
na prvoj godini - obrada foto-
grafija u Photoshop-u, crtajnje
vektorske grafike u Illustrator-u
ili na drugoj godini - Flash animacije
na treoj godini - Dreamweaver i
postavljanje na Internet.
Na taj nain bi uenicima bilo
ili slika, format teksta itd.) tj. odvaja izgled stranice od neuporedivo interesantnije i bili
njenog sadraja. XHTML je posljednja, standardizovana bi blii svjetskim tokovima informacionih tehnologija.
verzija HTML-a (HyperText Markup Language). On je Adobe-ov paket preporuuje njegova upotrebljivost,
sama osnova svjetskog Web-a. Prevod naziva govori o softverska kompatibilnost i rasprostranjenost meu
njegovoj funkciji - koristi se za formatiranje hiperteksta. korisnicima
Ove dvije tehnologije ine samu osnovu Web-a, ali svakako Ono to se zasad moe nauiti na asovima Web
ne otvaraju njihovom poznavaocu sve mogunosti. prezentacija je sama osnova, ali je potrebno dosta
Moderni Web sajtovi obuhvataju tehnologije za izradu nadgradnje znanja da bi se nivo mogao dovesti do
animacija, dinamike, interaktivnosti sa korisnikom, baza profesionalno upotrebljivog. Sa druge strane, ako ste
podataka. Tako su u stalnoj upotrebi Flash animacije, PHP zainteresovani, moete obratiti panju na ve pomenute
Web aplikacije, SQL baze podataka... tehnologije preko drugih kanala. U sljedeem broju
Jedan Web developer trebalo bi da nastavi izuavanje asopisa bie vie govora o nekoj od njih...
kroz upoznavanje sa ve pomenutim PHP-om (Hypertext
PreProcessor). Ovaj jezik koristi se za izradu dinamike Matija Tapukovi II-2
Web sajta i interaktivnosti sa korisnikom. Njegova

62
Rjeenja

Rjeenja
Zanimljiva matematika Matematiko-logike zagonetke i problemi

Ljepljivi smajliji: 1. Euro nije izgubljen; kelnerova 2 eura ne treba dodavati


iznosu od 12 eura ve oduzeti od ovog iznosa da bismo dobili
10 poena M I I I 10 eura, koliko iznosi raun. Situacija je veoma jasna: kod
blagajnika je ostalo 10 eura (iznos rauna), kod kelnera 2 eura
13 poena M I I B B
i po 1 euro kod gostiju, to iznosi tano 15 eura.
17 poena M M M B B B 2. Podjela nije izvrena pravilno, niti je to mogue uiniti.
1 1 1 17 34
Jedno rjeenje koje pokazuje koliko je smajlija svakog igraa Naime, kako je + + = = , dakle, manje od je-
palo u svaki red. Pronaite i druga. 2 3 9 18 36
dnog cijelog, pri podjeli automobila javio bi se ostatak.
Krivudavi zatvor: Brz nain kako odrediti da li se mrav nalazi Zbog toga je podjela prema testamentu nemogua uprkos
unutar ili izvan krivudavog zatvora je da se prebroji koliko snalaljivosti matematiara.
puta zamiljena linija, koja kree od mrava ka spoljanosti, Pozajmljivanje jednog automobila ima za posljedicu da je
sijee krivudavu liniju zatvora. Ako linija sijee krivu paran svako od brae dobio vie jer je:
broj puta, mrav se nalazi izvan zatvora; ako je u pitanju
35 35 35
neparan broj puta, onda je mrav unutar krivudavog zatvora. 18 > , 6> , 2> .
2 3 9
Elegantne elipse: Na slici je prikazano jedno rjeenje. Koliko bi
3. Oznaimo sa I istinito a sa L lano tvrenje. Mogue su etiti
tee bilo da smo dodali vie elipsi ili vie linija koje dijele.
kombinacije: II, IL, LI i LL. Prvu kombinaciju treba odbaciti
Kako bi kompjter rijeio ovaj problem?
jer je reeno da je bar jedna izreka lana. Druga i trea kombi-
nacija takoe ne zadovoljavaju uslove jer u oba sluaja, ako
jedno dijete lae, drugo dijete ne moe govoriti istinu. Prema
tome, oba djeteta lau. Dakle, djeak ima riu, a djevojica
crnu kosu
4. Ukljuite prvi prekida i ostavite ga da radi 5 minuta (koliko
je potrebno da se dobro zagrije sijalica). Ugasite ga i ukljuite
drugi prekida. Uete u sobu. Ako sijalica svijetli, onda drugi
prekida pali sijalicu. Ako ne svijetli, pipnete sijalicu. Ako je
sijalica topla, onda prvi prekida pali sijalicu, ako nije onda
je to trei prekida.
Tri kvadrata: Evo jednog rjeenja 5. Lako je provjeriti da nijedan trocifren broj koji se moe
napisati pomou cifara 3, 1 i 6 (njih ukupno est), nije djeljiv
1 2 8 9 6 4 sa 7
10 11 12 5 7 3 Rjeenje je vrlo prosto i duhovito (sjetite se, radi se o
gimnastiarima). Potrebno je da gimnastiar sa brojem 6 dubi
na glavi (tako se dobija broj 9), a da pored njega stanu (ili jo
Moete li pronai druga ?
bolje, kleknu) gimnastiari sa brojevima 3 i 1. Dobijeni broj
Tajanstveni niz: U pitanju je broj 7. Rjeenje je povezano sa 931 djeljiv je s 7.
brojem segmenata na standardnom ekranu digitrona koji
prikazuje cifre poev od 0. Sofizmi
Naite uljeza Dijeljenje nulom! S obzirom na to da je A = B, to je A B = 0,
pa ne moemo dijeliti sa A B
Uljez je broj 54
Svi ostali su za jedan manji od kvadratnog korijena nekog Sudoku
broja (4, 9, 16, 25, 36, 49, 64) Laka Tea
Iz jednog poteza 8 3 9 1 5 2 7 4 6 8 3 1 9 7 2 5 6 4
6 1 4 7 3 8 9 5 2 6 5 2 4 3 8 9 1 7
Krenuti iz tjemena pravog ugla. 5 2 7 6 4 9 3 1 8 9 4 7 5 6 1 2 8 3
3 8 6 4 9 5 2 7 1 4 7 9 2 1 6 3 5 8
Datum 1 4 5 2 7 3 8 6 9 3 1 8 7 4 5 6 2 9
Ako parne datume, kada je bio utorak oznaimo sa 7 9 2 8 1 6 5 3 4 5 2 6 8 9 3 7 4 1
9 7 1 3 2 4 6 8 5 2 8 4 3 5 9 1 7 6
x, x + 14 i x + 28, onda je x = 2, pa su dani 2, 16 i 30. 2 6 3 5 8 1 4 9 7 7 6 3 1 2 4 8 9 5
Poslednji petak je bio 26. dan u mjesecu. 4 5 8 9 6 7 1 2 3 1 9 5 6 8 7 4 3 2

63
Rjeenja
ibice metara, tj. stii e do drugog kraja daske. Za isto to vrijeme
ovjek, ija je brzina dvaput vea, prei e 20 metara.
1.
3. Crvenu, jer se frekvencija svjetlosti ne mijenja.

2. Okrenuti jednainu naopako. 4. U ovom sluaju nije eksplicitno navedeno da se otpor vazduha
moe zanemariti, pa je kinetika energija lopte na kraju leta
3.
manja nego na poetku. Takoe, na bilo kojoj visini, brzina
lopte je manja pri sputanju nego pri podizanju. Odavde je
jasno da je vrijeme penjanja krae od vremena sputanja.

4. 5. Kondukter i putnik koji je itavo vrijeme sjedio preli su


jednake puteve u odnosu na zemlju.

6. Lik koji se dobija poslije odbijanja zraka od oba ogledala,


poklapa se sa samim izvorom.
Plava tenost
7. Svrha nepokretnog kotura je samo da promijeni smjer sile.
Da bismo podigli teret teine Q, potrebno je da vuemo silom
40 stepeni Celzijusove skale jednako je 40 stepeni
F1= 100N prema dolje (slika A).
Farenhajtove skale. Konverzija iz C u F se dobija kada se
vrijednost pomnoi sa 9/5 i doda 32. Konverzija iz F u C Pokretni kotur djelovanje teine tereta prepolovi na obje
obrnuto, dobija se kada se oduzme 32, zatim razultat pomnoi strane ueta (slika B). Slijedi, pokretni kotur smanji silu
sa 5/9: Q
potrebnu za dizanje tereta na polovinu: F2 = = 50 N.
( 40 C) 9/5 = 72 + 32 = = 40 F, 2
slino 40 F 32 = (72) 5/9 = 40 C. F2
Zanimljiva fizika - pitanja i odgovori 10 + 10

1. Na prvi pogled, pozitivan odgovor bi protivrjeio Kulonovom


zakonu. Ali, prisjetimo se da naelektrisana tijela privlae
nenaelektrisana (na primjer, komadie papira privlai
naelektrisani ealj), a to takoe formalno ne odgovara Q F1 Q
Kulonovom zakonu. Ovakvo privlaenje objanjava se
preraspodjelom naelektrisanja u tijelu (sl. 1). Slika A Slika B

S druge strane, da bi se teret podigao za visinu H, kraj ueta


treba vui za dvostruko veu duinu (2H).

8. Sunce nam je najblie u zimskom solsticijumu (21.


decembra), a najudaljenije u ljetnjem solsticijumu (21.
Slika 1 Slika 2
juna).Vie temperature ljeti uslovljene su time to u ljetnjem
solsticijumu sunevi zraci padaju pod najveim uglom, a u
Ovaj efekat moe dovesti do toga da se privlae i istoimeno
zimskom pod najmanjim uglom prema naem horizontu.
naelektrisana tijela: blia strana jednog tijela moe izmijeniti
znak naelektrisanja (sl. 2). To je mogue kada se tijela nalaze
9. Uite u lift, stanite na vagu i posmatrajte teinu koju
dovoljno blizu jedno drugom i kada je naelektrisanje jednog vaga pokazuje kada liftom krenete vertikalno nanie s
od tijela mnogo vee od naelektrisanja drugog. U tom sluaju odgovarajuim ubrzanjem. Naravno, ovdje nije rije o
tijelo s manjim naelektrisanjem, opisanom preraspodjelom smanjenju mase tijela, nego o teini kao sili kojom pritiskamo
moe dovesti do privlaenja naelektrisanja suprotnog znaka. vagu.
2. Oznaimo sa A duinu daske, a sa M rastojanje koje pree
10. Polovina energije E prela je u unutranju energiju. Ne
osa cilindra do trenutka kada drugi kraj daske doe do njega.
moe se zanemariti sila trenja, bez obzira na njen intenzitet.
Poto je brzina daske u odnosu na podlogu dvaput vea od
Pri sputanju tijela u sistemu dolazi do oscilacija: tijelo po
brzine ose cilindra, onda je A + M = 2M, tj. M = A Prema
inerciji prolazi ravnoteni poloaj, prolazi nanie, zatim se
tome, ovjek treba da pree 20 metara.
vraa navie i, da nema trenja, ovo oscilovanje se nikad ne bi
Zadatak se moe elegantnije rijeiti u sistemu referencije zavrilo. (Smanjenjem sile trenja ne bismo mogli da utiemo
vezanom za dasku. U tom sistemu osa cilindra se kree na gubitak energije, ve bismo samo usporili proces dolaska
prema ovjeku brzinom istog intenziteta kao i u sistemu ovog sistema do ravnotenog poloaja.)
vezanom za podlogu i za odreeno vrijeme prei e 10

64

Das könnte Ihnen auch gefallen